Appearance
❓:Find the maximum value of M =frac{x}{2x + y} +frac{y}{2y + z}+frac{z}{2z + x} , x,y, z > 0
🧠:[<think>Alright, so I need to find the maximum value of M, which is defined as the sum of three fractions: x over (2x + y), y over (2y + z), and z over (2z + x), where x, y, and z are all positive real numbers. Hmm, okay. Let me start by understanding the problem better.First, the expression for M is a cyclic sum. Each term has a numerator that's one variable and a denominator that's twice that variable plus the next variable in the cycle. So, x is followed by y, y by z, and z by x. Since all variables are positive, all denominators are positive, so each term of M is a positive fraction less than 1/2? Wait, let me check that.Take the first term: x/(2x + y). Since x and y are positive, the denominator is greater than 2x, so the fraction is less than x/(2x) = 1/2. Similarly, each term is less than 1/2, so the sum M is less than 3*(1/2) = 3/2. But is 3/2 achievable? Probably not, because if you set y, z to be very small, say approaching zero, then x/(2x + y) approaches 1/2, y/(2y + z) approaches 0, and z/(2z + x) approaches 0. So in that case, M approaches 1/2. Similarly, if two variables are small, the maximum you can get from one term is 1/2, but the other terms might be negligible. Wait, but maybe there's a balance where each term is a bit more than 1/3? Hmm, not sure.Alternatively, maybe equality occurs when x = y = z? Let me test that. Suppose x = y = z = k, for some positive k. Then each term becomes k/(2k + k) = k/(3k) = 1/3. Therefore, M would be 1/3 + 1/3 + 1/3 = 1. So when all variables are equal, M equals 1. But earlier, I thought that if one variable is large compared to the others, maybe M can approach 1/2. But actually, when one variable is large, the other terms might not necessarily be negligible. Let me see.Suppose x is very large, and y and z are small. Then, x/(2x + y) ≈ x/(2x) = 1/2. Then y/(2y + z) ≈ y/z, but if y and z are both approaching zero but in some ratio. Similarly, z/(2z + x) ≈ z/x, which would approach zero. So if y and z are both approaching zero, but if y is much larger than z, then y/z might be significant. Wait, but if both y and z approach zero, but their ratio is fixed. Let's say y = a and z = b, where a and b are approaching zero, but their ratio is fixed. Then, the second term becomes a/(2a + b) and the third term is b/(2b + x). But if x is approaching infinity, then the third term becomes b/(2b + x) ≈ 0. So the second term is a/(2a + b). If we fix the ratio between a and b, say a = kb for some constant k. Then, the second term becomes kb/(2kb + b) = kb/(b(2k +1)) ) = k/(2k +1). So if we let k vary, this term can be up to 1/2 when k approaches infinity (i.e., a is much larger than b). But even then, the second term approaches 1/2, so total M would approach 1/2 + 1/2 = 1? Wait, but in reality, if x is very large, y is also very large but z is very small. Wait, maybe this approach is confusing. Let me try specific numbers.Let’s take x = 1000, y = 1000, z = 1. Then:First term: 1000/(2000 + 1000) = 1000/3000 = 1/3 ≈ 0.333Second term: 1000/(2000 + 1) ≈ 1000/2001 ≈ 0.49975Third term: 1/(2 + 1000) = 1/1002 ≈ 0.000999So total M ≈ 0.333 + 0.49975 + 0.000999 ≈ 0.83375. Hmm, that's higher than 1? Wait, no, that's less than 1. Wait, but when x = y = z, we have M = 1, which is higher. So in this case, even if two variables are large and one is small, the total is still less than 1. Interesting.Wait, another test case: x = 1, y = 1, z = 1: M = 1/3 + 1/3 + 1/3 = 1. If x = 1, y = 0.1, z = 0.1. Then:First term: 1/(2 + 0.1) ≈ 1/2.1 ≈ 0.476Second term: 0.1/(0.2 + 0.1) = 0.1/0.3 ≈ 0.333Third term: 0.1/(0.2 + 1) = 0.1/1.2 ≈ 0.083Total ≈ 0.476 + 0.333 + 0.083 ≈ 0.892. Still less than 1. Hmm.Another case: Let’s set x, y, z such that each denominator is minimized. Wait, but denominators have different variables, so it's tricky. Alternatively, maybe use the Cauchy-Schwarz inequality? Or the method of Lagrange multipliers? Since we need to maximize M with x, y, z > 0.Let me try Lagrange multipliers. Let’s set up the function:M = x/(2x + y) + y/(2y + z) + z/(2z + x)We need to maximize M. Let's compute partial derivatives with respect to x, y, z, set them equal to zero.First, derivative of M with respect to x:dM/dx = [ (1)(2x + y) - x(2) ] / (2x + y)^2 + [ derivative of the third term with respect to x ]Wait, third term is z/(2z + x). So derivative is -z/(2z + x)^2.So:dM/dx = [ (2x + y - 2x) ] / (2x + y)^2 - z/(2z + x)^2Simplify numerator of first term: y / (2x + y)^2 - z/(2z + x)^2.Similarly, derivative with respect to y:dM/dy = [ derivative of first term ] + [ derivative of second term ] + [ derivative of third term ]First term: x/(2x + y). Derivative is -x/(2x + y)^2.Second term: y/(2y + z). Derivative is [1*(2y + z) - y*2]/(2y + z)^2 = (2y + z - 2y)/(2y + z)^2 = z/(2y + z)^2.Third term: z/(2z + x). Derivative is 0 with respect to y.So total dM/dy = -x/(2x + y)^2 + z/(2y + z)^2.Similarly, derivative with respect to z:dM/dz = derivative of first term (0) + derivative of second term (-y/(2y + z)^2) + derivative of third term [1*(2z + x) - z*2]/(2z + x)^2 = (2z + x - 2z)/(2z + x)^2 = x/(2z + x)^2.So total dM/dz = -y/(2y + z)^2 + x/(2z + x)^2.At the maximum, the partial derivatives are zero. So we have the system:1. y/(2x + y)^2 - z/(2z + x)^2 = 02. -x/(2x + y)^2 + z/(2y + z)^2 = 03. -y/(2y + z)^2 + x/(2z + x)^2 = 0Hmm, this seems complex. Let me see if we can find symmetry here. Suppose that x = y = z. Then let's check if the partial derivatives are zero.Let x = y = z = k.Then, first equation:k/(2k + k)^2 - k/(2k + k)^2 = 0. Which is 0. Similarly, the second equation:- k/(2k + k)^2 + k/(2k + k)^2 = 0. Also zero. Third equation: same. So x = y = z is a critical point. So that's good. But we need to check if it's a maximum.But earlier, when we tested x = y = z, we got M = 1. But when we tested other values, we got lower M. So maybe this is the maximum. But we need to confirm that there are no other critical points where M is higher.Alternatively, perhaps we can use the Cauchy-Schwarz inequality or some other inequality. Let's think.Looking at each term, like x/(2x + y). Maybe we can manipulate this expression. Let me try to write each term as follows:x/(2x + y) = 1/(2 + y/x)Similarly, y/(2y + z) = 1/(2 + z/y)And z/(2z + x) = 1/(2 + x/z)So M = 1/(2 + y/x) + 1/(2 + z/y) + 1/(2 + x/z)Let me set variables a = y/x, b = z/y, c = x/z. Then, since a*b*c = (y/x)*(z/y)*(x/z) = 1. So the product of a, b, c is 1. Then, M can be written as:M = 1/(2 + a) + 1/(2 + b) + 1/(2 + c), where abc = 1.So the problem reduces to maximizing M = sum_{cyc} 1/(2 + a) with abc = 1 and a, b, c > 0.This substitution might make it easier to apply inequalities. Since abc = 1, we can let a = p, b = q, c = 1/(pq). Then, M = 1/(2 + p) + 1/(2 + q) + 1/(2 + 1/(pq)). But that seems more complicated. Maybe instead, use the method of substitution with symmetry.Alternatively, if we let a = b = c = 1, then M = 3*(1/3) = 1, which is the case when x = y = z. So perhaps this is the maximum. To verify, we can try using the AM-GM inequality or convexity.Let’s analyze the function f(t) = 1/(2 + e^t), since if we set a = e^t, then it's similar. Wait, maybe not. Alternatively, consider the function f(a) = 1/(2 + a). Since f''(a) = 2/(2 + a)^3 > 0 for a > 0, the function is convex. Wait, f(a) = 1/(2 + a), first derivative f’(a) = -1/(2 + a)^2, second derivative f''(a) = 2/(2 + a)^3, which is positive for a > -2. Since a > 0, f is convex on its domain. Therefore, by Jensen's inequality, we have:f(a) + f(b) + f(c) >= 3f( (a + b + c)/3 )But since abc = 1, the arithmetic mean (a + b + c)/3 >= (abc)^{1/3} = 1. So, 3f( (a + b + c)/3 ) <= 3f(1) = 3*(1/3) = 1. But since f is convex, Jensen gives us that the sum f(a) + f(b) + f(c) >= 3f(AM). But since the inequality goes the other way, maybe this approach isn't helpful. Wait, hold on. If f is convex, then:( f(a) + f(b) + f(c) ) / 3 >= f( (a + b + c)/3 )But since (a + b + c)/3 >=1, and f is decreasing, then f( (a + b + c)/3 ) <= f(1). So:( f(a) + f(b) + f(c) ) /3 >= f( (a + b + c)/3 ) <= f(1)Therefore,( f(a) + f(b) + f(c) ) >= 3 f( (a + b + c)/3 ) >= 3 f(1) ?Wait, no. If f is convex and decreasing, then f( (a + b + c)/3 ) <= ( f(a) + f(b) + f(c) ) /3. Because for convex functions, the function evaluated at the average is less than or equal to the average of the function. So:f( (a + b + c)/3 ) <= ( f(a) + f(b) + f(c) ) /3But since f is decreasing, and (a + b + c)/3 >=1, then:( f(a) + f(b) + f(c) ) /3 >= f( (a + b + c)/3 ) >= f( something >=1 )But since f is decreasing, f(1) = 1/3, and if the average (a + b + c)/3 is greater than or equal to 1, then f( (a + b + c)/3 ) <= f(1) = 1/3. Therefore,( f(a) + f(b) + f(c) ) /3 >= f( (a + b + c)/3 ) <= 1/3Which gives:( f(a) + f(b) + f(c) ) >= 3*(something <=1/3 )But this seems to not directly give us the bound we need.Alternatively, perhaps use the substitution a = 1/p, b = 1/q, c = 1/r, such that p*q*r = 1. Wait, not sure.Alternatively, note that when abc = 1, we can set a = x/y, b = y/z, c = z/x, but maybe this is not helpful.Wait, maybe use Lagrange multipliers on the transformed variables. Let’s define the problem as maximize M = 1/(2 + a) + 1/(2 + b) + 1/(2 + c) subject to abc = 1.Let’s set up the Lagrangian: L = 1/(2 + a) + 1/(2 + b) + 1/(2 + c) - λ(abc -1 )Take partial derivatives:dL/da = -1/(2 + a)^2 - λ bc = 0Similarly, dL/db = -1/(2 + b)^2 - λ ac = 0dL/dc = -1/(2 + c)^2 - λ ab = 0And the constraint abc =1.So, from the first three equations:-1/(2 + a)^2 = λ bc-1/(2 + b)^2 = λ ac-1/(2 + c)^2 = λ abSince abc =1, we can express bc = 1/a, ac =1/b, ab =1/c. Substitute into the equations:-1/(2 + a)^2 = λ (1/a)-1/(2 + b)^2 = λ (1/b)-1/(2 + c)^2 = λ (1/c)So, we have:λ = -a/(2 + a)^2 = -b/(2 + b)^2 = -c/(2 + c)^2Therefore, we get that:a/(2 + a)^2 = b/(2 + b)^2 = c/(2 + c)^2So, the variables a, b, c must satisfy this equality. Let’s denote k = a/(2 + a)^2 = b/(2 + b)^2 = c/(2 + c)^2So, we need to solve for a, b, c >0 such that a/(2 + a)^2 = b/(2 + b)^2 = c/(2 + c)^2 and abc =1.Assume that a = b = c. Then, since abc =1, a^3 =1 => a=1. Let's check if this satisfies the equation:1/(2 +1)^2 =1/9. So, k=1/9. Then, if a =1, then indeed, a/(2 +a)^2=1/9. So a=b=c=1 is a solution. Therefore, this gives M = 1/(2 +1)+1/(2 +1)+1/(2 +1)=1. Now, we need to check if there's another solution where a, b, c are not all equal.Suppose there exists another solution where two variables are equal, say a = b ≠ c. Then, from a/(2 +a)^2 = c/(2 +c)^2 and abc =1. Let me set a = b, so a^2 c =1 => c =1/a^2. Then, the equation a/(2 +a)^2 = c/(2 +c)^2 becomes:a/(2 +a)^2 = (1/a^2)/(2 +1/a^2)^2Multiply both sides by a^2*(2 +1/a^2)^2:a^3/(2 +a)^2 = 1/(2 +1/a^2)^2Let’s simplify both sides. Let me compute left-hand side (LHS) and right-hand side (RHS):LHS = a^3/(2 + a)^2RHS = 1/(2 + 1/a^2)^2 = 1/( (2a^2 +1)/a^2 )^2 = (a^2)^2/(2a^2 +1)^2 = a^4/(2a^2 +1)^2Set LHS = RHS:a^3/(2 + a)^2 = a^4/(2a^2 +1)^2Cancel a^3 from both sides:1/(2 +a)^2 = a/(2a^2 +1)^2Cross-multiplied:(2a^2 +1)^2 = a(2 +a)^2Expand both sides:Left side: (2a^2 +1)^2 =4a^4 +4a^2 +1Right side: a(4 +4a +a^2) =4a +4a^2 +a^3Set equal:4a^4 +4a^2 +1 =4a +4a^2 +a^3Simplify:4a^4 -a^3 +0a^2 -4a +1 =0So,4a^4 -a^3 -4a +1 =0This is a quartic equation. Let me try to factor it.Let’s try rational roots. Possible rational roots are ±1, ±1/2, ±1/4.Test a=1:4(1)^4 -1^3 -4(1) +1 =4 -1 -4 +1=0. So a=1 is a root.Therefore, we can factor (a -1) from the quartic.Using polynomial division or synthetic division:Divide 4a^4 -a^3 -4a +1 by (a -1):Coefficients: 4, -1, 0, -4, 1Bring down 4. Multiply by 1:4. Add to -1:3.Multiply by1:3. Add to 0:3.Multiply by1:3. Add to -4:-1.Multiply by1:-1. Add to1:0.So quotient polynomial is 4a^3 +3a^2 +3a -1.Thus, 4a^4 -a^3 -4a +1 = (a -1)(4a^3 +3a^2 +3a -1)Now, solve 4a^3 +3a^2 +3a -1=0.Again, test rational roots. Possible roots:1, -1, 1/2, -1/2, 1/4, -1/4.Test a=1/2:4*(1/8) +3*(1/4) +3*(1/2) -1 =0.5 +0.75 +1.5 -1=1.75≠0Test a=1/4:4*(1/64) +3*(1/16) +3*(1/4) -1=0.0625 +0.1875 +0.75 -1≈0.0625+0.1875=0.25; 0.25+0.75=1; 1 -1=0. So a=1/4 is a root.Thus, factor (a -1/4) from 4a^3 +3a^2 +3a -1.Use synthetic division:Divide 4a^3 +3a^2 +3a -1 by (a -1/4).Wait, but coefficients are 4,3,3,-1. Root at a=1/4.Multiply through by 4 to make it integer coefficients. Let me write polynomial as 4a^3 +3a^2 +3a -1. Let x = a.Evaluate at x=1/4: 4*(1/64) +3*(1/16) +3*(1/4) -1= (1/16)+(3/16)+(3/4)-1= (4/16)+(12/16)-1= (16/16)-1=0. So correct.Use synthetic division with root 1/4:Coefficients:4,3,3,-1Multiply by 1/4:Start with 4Bring down 4. Multiply by1/4:1. Add to3:4.Multiply by1/4:1. Add to3:4.Multiply by1/4:1. Add to-1:0.So quotient is 4a^2 +4a +4. Therefore,4a^3 +3a^2 +3a -1=(a -1/4)(4a^2 +4a +4)Thus, original quartic factors as:(a -1)(a -1/4)(4a^2 +4a +4)The quadratic 4a^2 +4a +4 has discriminant 16 -64= -48 <0, so no real roots.Thus, real roots are a=1 and a=1/4.Thus, going back, when we assumed a =b, the equation reduces to solutions at a=1 and a=1/4. Let's check these.Case 1: a=1. Then, since a =b=1, then c=1/a^2=1. So this is the symmetric case.Case 2: a=1/4. Then, since a=1/4, then c=1/a^2=16. So c=16. Then, from the equation a/(2 +a)^2 = c/(2 +c)^2, check:a=1/4: left-hand side (1/4)/(2 +1/4)^2=(1/4)/(9/4)^2=(1/4)/(81/16)= (1/4)*(16/81)=4/81≈0.04938c=16: right-hand side 16/(2 +16)^2=16/324=4/81≈0.04938. So equal. Thus, this is a valid solution.Therefore, there is another critical point where a =b=1/4, c=16. Similarly, permutations. So, we have asymmetric critical points.Now, compute M for a=1/4, b=1/4, c=16.So M =1/(2 +1/4)+1/(2 +1/4)+1/(2 +16)=1/(9/4)+1/(9/4)+1/18=4/9 +4/9 +1/18=8/9 +1/18=17/18≈0.9444. Which is less than 1. Similarly, other permutations would give the same value.Therefore, the maximum at symmetric point a=b=c=1 gives M=1, while the asymmetric critical points give lower M. Therefore, likely that M=1 is the maximum.But just to confirm, let's check another possible case where two variables are different. Wait, but we just considered the case when two variables are equal and found that the asymmetric solution gives a lower M. Maybe the maximum occurs only at the symmetric point.Alternatively, let's check another critical point where a, b, c are different. But given the complexity, maybe it's difficult. Alternatively, consider the second derivative test, but that might be too involved.Alternatively, since we found that at a=b=c=1, the value is 1, which is higher than other tested cases and the asymmetric critical points, and given the symmetry of the problem, it's reasonable to conjecture that M=1 is the maximum, achieved when x=y=z.But let me test another case where variables are not equal but set to make the denominators equal.Suppose 2x + y = 2y + z = 2z + x = k for some k. Then:From 2x + y = k and 2y + z = k and 2z + x =k.Solve this system.From first equation: y =k -2xSecond equation: z =k -2y =k -2(k -2x)=k -2k +4x= -k +4xThird equation: 2z +x =k => 2*(-k +4x) +x =k => -2k +8x +x =k =>9x=3k =>x= (3k)/9=k/3Thus, x= k/3. Then, y =k -2*(k/3)=k -2k/3=k/3. Similarly, z= -k +4*(k/3)= -k +4k/3= k/3. Thus, x=y=z=k/3. Therefore, the only solution where denominators are equal is x=y=z. So in that case, M=1. So this supports that x=y=z is the optimal.Alternatively, consider the following approach. Let me use the Cauchy-Schwarz inequality on each term.Note that for each term, we have:x/(2x + y) = x/(x + x + y). Maybe we can compare this to some average. Let me recall that for positive numbers a, b, c, the fraction a/(a + b + c) is less than 1/2 if a < b + c. But not sure.Alternatively, use the following substitution: Let’s set t = y/x, s = z/y, r = x/z. Then, t*s*r =1. Then, as before, M becomes 1/(2 + t) +1/(2 + s) +1/(2 + r). But this seems similar to previous substitution.Alternatively, note that for each term, x/(2x + y) <= x/(2x + 2√(xy)) by AM-GM on y and 0. Wait, but AM-GM on y and what? Wait, maybe not.Alternatively, use the substitution u = y/x, v = z/y, w =x/z, as before, which leads to u*v*w=1. Then, M =1/(2 + u) +1/(2 +v ) +1/(2 +w ). With u*v*w=1. Let’s set u = p, v = q, so w=1/(p*q). Then, M =1/(2 +p) +1/(2 + q) +1/(2 +1/(p*q)). To maximize this.Take partial derivatives with respect to p and q and set to zero. But this might be complicated. Alternatively, consider that maximum occurs when p=q=1. Which gives M=1. If we take p=1, q=1, then M=1/(3)+1/(3)+1/(3)=1. If we take p=2, q=2, then w=1/4, so M=1/(4)+1/(4)+1/(2 +1/4)=0.25+0.25+4/9≈0.5+0.444≈0.944, which is less than 1.Alternatively, take p approaching 0, then w approaches infinity. Then, first term approaches 1/2, second term approaches 1/(2 + q), third term approaches 0. So total M approaches 1/2 +1/(2 + q). To maximize this, set q approaching 0, so M approaches 1/2 +1/2=1. Wait, so if p approaches 0 and q approaches 0, then w approaches infinity, but M approaches 1. Wait, but if p and q approach 0, then u approaches 0, v approaches 0, w approaches infinity. Let me check with substitution.If u approaches 0, then y/x approaches 0, so y approaches 0. Then, x is fixed? Wait, variables are positive, but we can scale them. Wait, if u = y/x approaches 0, then y is much smaller than x. Similarly, v = z/y approaches 0, so z is much smaller than y. Then w =x/z = (x/y)*(y/z)= (1/u)*(1/v). If u and v approach 0, then w approaches infinity. Then, the terms:1/(2 + u) ≈1/2,1/(2 + v) ≈1/2,1/(2 + w)≈0.So total M≈1/2 +1/2 +0=1. But this is the same as the symmetric case. However, this is in the limit as variables become very small. Wait, but how does that translate back to x, y, z?If u approaches 0, meaning y approaches 0 with x fixed. Then v = z/y approaches 0, so z approaches 0 even faster. Then w =x/z approaches infinity. So x is fixed, y and z approach 0. Then, compute M:x/(2x + y) ≈x/(2x)=1/2,y/(2y + z)≈ y/z. If y and z approach 0 with y/z approaching 0, then this term approaches 0,z/(2z +x)≈ z/x≈0.Thus, total M approaches1/2, which contradicts the previous thought. Wait, so there's a discrepancy here. On one hand, substituting u, v, w approaching 0,0,infinity gives M approaching1, but in reality, with x fixed and y, z approaching 0, M approaches1/2. Therefore, the substitution approach might have a different scaling.Wait, maybe when u and v approach 0, the variables x, y, z are scaled accordingly. Let me think. If u = y/x approaches 0, then y = u*x. If v = z/y approaches 0, then z = v*y = v*u*x. Therefore, z = u*v*x. Then, w =x/z =1/(u*v). If u and v approach 0, then w approaches infinity. So in terms of x, y, z:x =x,y= u*x,z= u*v*x.If we fix x, then y and z approach 0 as u and v approach 0. Therefore, M:x/(2x + y)=x/(2x +u x)=1/(2 +u) ≈1/2,y/(2y + z)=u x/(2u x +u v x)=u/(2u +u v)=1/(2 +v)≈1/2,z/(2z +x)=u v x/(2 u v x +x)=u v/(2 u v +1)≈0 as u v approaches0.Therefore, M≈1/2 +1/2 +0=1. But in reality, when we fix x and send y,z to zero, the second term becomes y/(2y + z). If z is going to zero faster than y, say z = y^2, then as y approaches 0, y/(2y + y^2) ≈ y/(2y) =1/2. So then M≈1/2 +1/2 +0=1. But when I took x=1000, y=1000, z=1, which isn't the case where y,z approach zero. Wait, but when I set x=1000, y=1000, z=1, which isn't the case where y and z approach zero. Maybe if I set x=1, y=ε, z=ε^2 with ε approaching zero, then:First term:1/(2 + ε) ≈1/2,Second term:ε/(2ε +ε^2)≈ε/(2ε)=1/2,Third term:ε^2/(2ε^2 +1)≈0.So total M≈1/2 +1/2 +0=1.But when I earlier set x=1000, y=1000, z=1, which is different. But in the case where variables are scaled such that y and z approach zero in a certain way, M approaches1. However, when I tried specific numbers with x=1000, y=1000, z=1, M≈0.833. So in some cases, M approaches1, but in others, it approaches less.Wait, this is confusing. It seems that depending on the rate at which variables approach zero or infinity, M can approach1 or not. So, theoretically, is 1 the supremum? Because in some cases, we can make M arbitrarily close to1 by taking y and z approaching zero in such a way that y/z approaches0. But when we tried specific finite values, M was less than1. So, maybe the maximum value is1, but it's not achieved, but approached in the limit. But earlier, when we used Lagrange multipliers, we found a critical point at x=y=z, which gives M=1. So which is it? Is 1 achieved at x=y=z, and also approached in the limit as variables are scaled in a certain way, or is1 the maximum achieved at x=y=z?Wait, when x=y=z, M=1, which is a concrete value. But according to the limit analysis, M can approach1 as variables are scaled. So, perhaps1 is the maximum, achieved both at x=y=z and approached in the limit. However, in reality, for x,y,z>0, the maximum is achieved at x=y=z, since at the limit points, variables are approaching zero or infinity, but not actually reaching there. Therefore, the maximum value is1, achieved when x=y=z.But need to confirm this with more rigor. Let me try another approach. Let's use the substitution x = a, y = b, z = c. Then, the problem is cyclic. Let me consider the function f(a,b,c) = a/(2a + b) + b/(2b + c) + c/(2c + a). We can use the method of substitution by letting a = b = c. As shown, this gives f(a,a,a) =1. To check if this is the maximum, consider the following inequality:For each term, a/(2a + b) <=1/3*(some expression). Wait, maybe use the Cauchy-Schwarz inequality.Let’s consider the sum:Sum_{cyc} a/(2a + b) = Sum_{cyc} [a/(2a + b)]Let’s apply Cauchy-Schwarz in the form:Sum [a/(2a + b)] <= Sum [ (a)^2 / (2a^2 + ab) ]But not sure if helpful.Alternatively, use the following idea: For each term, a/(2a + b) =1/(2 + b/a). Let’s set t =b/a, u =c/b, v =a/c. Then, t*u*v =1. Then, M=1/(2 + t) +1/(2 + u) +1/(2 + v). But this is similar to before.Alternatively, use the substitution t =b/a, so a/b =1/t, similarly for others. But not helpful.Alternatively, consider the function f(t) =1/(2 + t), which is convex for t> -2. Since f''(t) = 2/(2 +t)^3 >0 for t> -2, which is true here as t =b/a>0.Thus, by Jensen’s inequality, for convex f:f(t) +f(u) +f(v) >=3f( (t +u +v)/3 )But since t*u*v=1, the arithmetic mean (t +u +v)/3 >=1, so f( (t +u +v)/3 ) <=f(1)=1/3. Hence,Sum f(t) >=3*(1/3)=1. Wait, that would imply that M >=1. But when variables are such that t =u =v=1, M=1. But if we have other values, M can be larger? Wait, but in our previous example, with a=1/4, b=1/4, c=16, M≈17/18<1, which contradicts this conclusion.Therefore, there must be a mistake here. Wait, Jensen’s inequality for convex functions states that the average of the function at some points is greater than or equal to the function at the average of the points. So,(f(t) +f(u) +f(v))/3 >=f( (t +u +v)/3 )Since f is convex. Therefore,Sum f(t) >=3f( (t +u +v)/3 )But since t +u +v >=3*(t u v)^{1/3}=3, and f is decreasing,f( (t +u +v)/3 ) <=f(1)=1/3Therefore,Sum f(t) >=3*(something <=1/3 )Thus,Sum f(t) >= something <=1But this doesn't give a useful lower bound. So, this approach is not helpful.Alternatively, reverse the inequality. Since f is convex, maybe use the inequality in the other direction. But f is convex, so can't get an upper bound from Jensen.Alternatively, use the Cauchy-Schwarz inequality. Let's write:Sum [ a/(2a +b) ] = Sum [ a^2/(2a^2 +ab) ]By Cauchy-Schwarz,[Sum (a^2/(2a^2 +ab)) ] >= [ (a +b +c)^2 ] / [Sum (2a^2 +ab) ]Compute denominator:Sum (2a^2 +ab) =2(a^2 +b^2 +c^2) +ab +bc +caThus,Sum [ a/(2a +b) ] >= (a +b +c)^2 / [2(a^2 +b^2 +c^2) +ab +bc +ca ]To find a lower bound, but we need an upper bound. So Cauchy-Schwarz might not be helpful here.Alternatively, try to find an upper bound. Let me consider the following approach: For each term, a/(2a +b) <= a/(2a +2√(ab)) by AM-GM, since b + something >=2√(ab). Wait, not exactly. Wait, 2a +b =a +a +b >=3*(a*a*b)^{1/3} by AM-GM, but not sure.Alternatively, use the substitution b =ka, c=mb, then z= mka, x =a. Then, variables expressed in terms of a, k, m. But this might complicate things.Alternatively, let’s use the method of substituting variables such that x + y + z =1. Since the expression is homogeneous. Wait, check homogeneity. Each term in M is degree 0: numerator is x, denominator is 2x + y, which is also degree 1. Therefore, M is homogeneous of degree0. Thus, we can assume WLOG that x + y + z =1. Then, maximize M under this constraint.But perhaps this helps. Let’s set x + y + z =1. Then, we can use Lagrange multipliers with this constraint. But this might not necessarily make it easier.Alternatively, consider testing more cases. For example, x = y = z gives M=1. If x=2, y=1, z=1. Then:First term:2/(4 +1)=2/5=0.4Second term:1/(2 +1)=1/3≈0.333Third term:1/(2 +2)=1/4=0.25Total M≈0.4 +0.333 +0.25≈0.983 <1.Another case: x=1, y=2, z=1.First term:1/(2 +2)=0.25Second term:2/(4 +1)=0.4Third term:1/(2 +1)=0.333Total≈0.25 +0.4 +0.333≈0.983 <1.Another case: x=1, y=0.5, z=0.5.First term:1/(2 +0.5)=1/2.5=0.4Second term:0.5/(1 +0.5)=0.5/1.5≈0.333Third term:0.5/(1 +1)=0.25Total≈0.4 +0.333 +0.25≈0.983 <1.Another test case: x=1, y=0.1, z=0.1.First term:1/2.1≈0.476Second term:0.1/0.3≈0.333Third term:0.1/1.1≈0.0909Total≈0.476 +0.333 +0.0909≈0.899 <1.So in all these cases, M is less than1. Only when variables are equal, we get M=1. In the limit where two variables approach zero in a specific way, M approaches1. But since variables must be positive, the maximum achieved value is1, attained when x=y=z.Therefore, after all these considerations and checks, I believe the maximum value of M is1, achieved when x, y, z are all equal.Final AnswerThe maximum value of ( M ) is boxed{1}.</think>To find the maximum value of ( M = frac{x}{2x + y} + frac{y}{2y + z} + frac{z}{2z + x} ) where ( x, y, z > 0 ), we start by analyzing the expression and testing various cases.1. Testing Equal Variables: - Suppose ( x = y = z ). Then each term becomes ( frac{x}{2x + x} = frac{1}{3} ). - Summing these terms gives ( M = frac{1}{3} + frac{1}{3} + frac{1}{3} = 1 ).2. Testing Extremes: - If one variable is very large compared to the others, the terms approach values less than ( frac{1}{2} ), leading to a total less than 1. - For example, setting ( x ) very large and ( y, z ) very small results in terms approaching ( frac{1}{2} ), 0, and 0, giving a total approaching ( frac{1}{2} ).3. Using Lagrange Multipliers: - Setting up the Lagrangian for the function ( M ) with constraint ( abc = 1 ) (after substituting variables) leads to the critical point ( a = b = c = 1 ), confirming ( M = 1 ).4. Convexity and Inequalities: - Using Jensen's inequality on the convex function ( f(t) = frac{1}{2 + t} ) and considering the constraint ( abc = 1 ) confirms that the maximum value is achieved when ( a = b = c = 1 ).5. Verification with Specific Cases: - Various specific cases (e.g., ( x = 1000, y = 1000, z = 1 )) result in values of ( M ) less than 1, reinforcing that the maximum value is achieved when ( x = y = z ).After thorough analysis and testing different scenarios, the maximum value of ( M ) is found to be 1, achieved when ( x = y = z ).[boxed{1}],<think>Okay, so I have this problem here: Find the maximum value of ( M = frac{x}{2x + y} + frac{y}{2y + z} + frac{z}{2z + x} ) where ( x, y, z > 0 ). Hmm, interesting. Let me try to figure this out step by step. First, I need to maximize this expression. It's a sum of three fractions, each involving two variables. The variables are all positive, which is good to note. Maybe symmetry can help here? But the denominators aren't symmetric in the same way. Let's see. Each term has a variable in the numerator and a denominator that's twice that variable plus another one. So, for the first term, it's ( x ) over ( 2x + y ), then ( y ) over ( 2y + z ), and ( z ) over ( 2z + x ). I wonder if there's a substitution or some inequality I can apply here. Since all variables are positive, maybe using the AM-GM inequality or Cauchy-Schwarz? Let me recall those. The AM-GM inequality states that the arithmetic mean is greater than or equal to the geometric mean. Cauchy-Schwarz is useful for sums of fractions, perhaps. Alternatively, maybe setting variables proportional to each other? Let me try some approaches.First, let's consider if there's symmetry when ( x = y = z ). Let me test that. If ( x = y = z = k ), then each term becomes ( frac{k}{2k + k} = frac{k}{3k} = frac{1}{3} ). Therefore, the total sum would be ( 3 times frac{1}{3} = 1 ). So, if all variables are equal, ( M = 1 ). But is this the maximum? Maybe not. Let's see if we can get a higher value by adjusting the variables.Suppose I set one variable very small compared to the others. Let's say ( x ) is very small. Then the first term becomes ( frac{x}{2x + y} approx frac{0}{0 + y} = 0 ). The last term is ( frac{z}{2z + x} approx frac{z}{2z} = frac{1}{2} ). The middle term is ( frac{y}{2y + z} ). If ( x ) is small, maybe ( y ) and ( z ) can be adjusted. Let's try specific numbers.Let me set ( x = 1 ), ( y = 1 ), and ( z = 1 ). Then, as before, each term is ( 1/3 ), sum is 1. If I set ( x = 1 ), ( y = 1 ), and ( z ) approaches 0. Then, the middle term ( frac{1}{2*1 + 0} = 1/2 ), the last term ( frac{0}{0 + 1} = 0 ), so total sum ( 1/(2 + 1) + 1/2 + 0 = 1/3 + 1/2 = 5/6 ≈ 0.833 ), which is less than 1. Hmm, that's lower.Wait, maybe I miscalculated. Wait, if ( z ) approaches 0, then the third term is ( z/(2z + x) approx 0/(0 + 1) = 0 ), the first term is ( x/(2x + y) = 1/(2 + 1) = 1/3 ), and the second term is ( y/(2y + z) = 1/(2 + 0) = 1/2 ). So total is 1/3 + 1/2 = 5/6, which is about 0.833. So lower than 1. What if I set one variable to be much larger? Let's say ( x ) is very large. Then the first term becomes ( x/(2x + y) ≈ 1/2 ). The last term is ( z/(2z + x) ≈ z/x ≈ 0 ). The middle term is ( y/(2y + z) ). If ( x ) is large, maybe ( y ) and ( z ) can be adjusted. Let's set ( x ) approaching infinity, ( y = 1 ), ( z = 1 ). Then first term ≈ 1/2, middle term ( 1/(2 + 1) = 1/3 ), last term ≈ 0. So total ≈ 1/2 + 1/3 = 5/6 again. Hmm, same as before.Alternatively, maybe set two variables small and one large. For example, let ( x ) and ( z ) be very small, and ( y ) large. Then first term ( x/(2x + y) ≈ 0 ), middle term ( y/(2y + z) ≈ 1/2 ), last term ( z/(2z + x) ≈ 0 ). So total ≈ 1/2. That's even worse. Hmm.So maybe making variables unequal doesn't help. Wait, but when all variables are equal, we get 1. What if variables are not equal but in some proportion?Let me try setting ( x = y = z ). Then M = 1 as before. Let me try another set. Suppose ( x = 2 ), ( y = 1 ), ( z = 1 ). Then first term: 2/(4 +1)= 2/5=0.4, second term:1/(2 +1)=1/3≈0.333, third term:1/(2 +2)=1/4=0.25. Total ≈0.4+0.333+0.25≈0.983. Less than 1.Another set: x=1, y=2, z=1. Then first term:1/(2 +2)=1/4=0.25, second term:2/(4 +1)=2/5=0.4, third term:1/(2 +1)=1/3≈0.333. Total ≈0.25+0.4+0.333≈0.983. Still less than 1.Hmm. Maybe trying different ratios. Let's try x=1, y=1, z=2. First term:1/(2 +1)=1/3≈0.333, second term:1/(2 +2)=1/4=0.25, third term:2/(4 +1)=2/5=0.4. Total≈0.333+0.25+0.4≈0.983. Same as before.Hmm. Maybe we need to test some other ratios. Let me try x=1, y=2, z=3. First term:1/(2+2)=1/4=0.25, second term:2/(4 +3)=2/7≈0.2857, third term:3/(6 +1)=3/7≈0.4286. Total≈0.25+0.2857+0.4286≈0.9643. Still less than 1.Wait, perhaps there's a better combination. Maybe setting variables in a cyclic pattern. For example, x=1, y=a, z=b. Maybe set a and b such that each term is maximized. But this might not be straightforward.Alternatively, maybe use calculus. Let's consider partial derivatives. Since M is a function of x, y, z. To find the maximum, set partial derivatives to zero. Let's attempt that.Let me denote ( M = frac{x}{2x + y} + frac{y}{2y + z} + frac{z}{2z + x} ).Compute partial derivative of M with respect to x:First term: derivative of ( frac{x}{2x + y} ) with respect to x is ( frac{(1)(2x + y) - x(2)}{(2x + y)^2} = frac{2x + y - 2x}{(2x + y)^2} = frac{y}{(2x + y)^2} ).Third term: derivative of ( frac{z}{2z + x} ) with respect to x is ( frac{0 - z(1)}{(2z + x)^2} = frac{ - z}{(2z + x)^2} ).The second term doesn't involve x, so derivative is zero. So overall:( frac{partial M}{partial x} = frac{y}{(2x + y)^2} - frac{z}{(2z + x)^2} ).Similarly, compute partial derivative with respect to y:First term: derivative of ( frac{x}{2x + y} ) with respect to y is ( frac{ - x }{(2x + y)^2} ).Second term: derivative of ( frac{y}{2y + z} ) with respect to y is ( frac{(1)(2y + z) - y(2)}{(2y + z)^2} = frac{2y + z - 2y}{(2y + z)^2} = frac{z}{(2y + z)^2} ).Third term doesn't involve y, so derivative is zero. So overall:( frac{partial M}{partial y} = frac{ - x }{(2x + y)^2} + frac{z}{(2y + z)^2} ).Similarly, partial derivative with respect to z:Second term: derivative of ( frac{y}{2y + z} ) with respect to z is ( frac{ - y }{(2y + z)^2} ).Third term: derivative of ( frac{z}{2z + x} ) with respect to z is ( frac{(1)(2z + x) - z(2)}{(2z + x)^2} = frac{2z + x - 2z}{(2z + x)^2} = frac{x}{(2z + x)^2} ).First term doesn't involve z, so derivative is zero. So overall:( frac{partial M}{partial z} = frac{ - y }{(2y + z)^2} + frac{x}{(2z + x)^2} ).At the maximum, all partial derivatives are zero. Therefore, we have the system of equations:1. ( frac{y}{(2x + y)^2} = frac{z}{(2z + x)^2} )2. ( frac{z}{(2y + z)^2} = frac{x}{(2x + y)^2} )3. ( frac{x}{(2z + x)^2} = frac{y}{(2y + z)^2} )Hmm, this system looks cyclic. Let me denote variables as x, y, z. Let's see if we can assume symmetry here, i.e., x = y = z. Let's check if that satisfies the equations.If x = y = z = k, then equation 1 becomes:Left side: ( frac{k}{(2k + k)^2} = frac{k}{9k^2} = frac{1}{9k} )Right side: ( frac{k}{(2k + k)^2} = frac{1}{9k} ). So they are equal. Similarly, equation 2:Left side: ( frac{k}{(2k + k)^2} = frac{1}{9k} )Right side: ( frac{k}{(2k + k)^2} = frac{1}{9k} ). Equal.Equation 3 similarly. So x = y = z is a critical point. But we need to check if this is a maximum. Since when we tried variables equal, we get M = 1, but when variables are unequal, we get lower values. Maybe this is the maximum. But how to confirm?Alternatively, maybe there are other critical points where variables are in different proportions. Let me try to explore the system of equations.From equation 1: ( frac{y}{(2x + y)^2} = frac{z}{(2z + x)^2} )From equation 2: ( frac{z}{(2y + z)^2} = frac{x}{(2x + y)^2} )From equation 3: ( frac{x}{(2z + x)^2} = frac{y}{(2y + z)^2} )Let me denote some ratios. Let me suppose that x = y = z. Then all equations hold. But if they aren't equal, maybe variables are in proportion. Let me set variables in a ratio. Let me suppose that x = a, y = b, z = c, and look for proportional relations.Alternatively, let me set variables such that 2x + y = 2y + z = 2z + x. Let's see if that's possible. Let me call all denominators equal to some constant k.So:2x + y = k2y + z = k2z + x = kSo we have a system of equations:1. 2x + y = k2. 2y + z = k3. 2z + x = kWe can solve this system. Let's express variables in terms of k.From equation 1: y = k - 2xFrom equation 2: z = k - 2y = k - 2(k - 2x) = k - 2k + 4x = -k + 4xFrom equation 3: 2z + x = k => 2(-k + 4x) + x = k => -2k + 8x + x = k => -2k + 9x = k => 9x = 3k => x = (3k)/9 = k/3Then x = k/3. Then y = k - 2*(k/3) = k - 2k/3 = k/3. Similarly, z = -k + 4x = -k + 4*(k/3) = -k + 4k/3 = k/3. Therefore, x = y = z = k/3. So the only solution is x = y = z. Therefore, if we set all denominators equal, variables must be equal. Therefore, this would lead us back to x = y = z, and M = 1. So maybe the maximum is indeed 1. But let me check another approach. Let's use the method of substitution. Let me set variables such that x = y = z. Then M = 1. Suppose we set variables in another ratio, say y = kx, z = mx. Then express everything in terms of x. Let's try.Let me set y = kx and z = mx, where k and m are positive constants. Then M becomes:First term: ( frac{x}{2x + y} = frac{x}{2x + kx} = frac{1}{2 + k} )Second term: ( frac{y}{2y + z} = frac{kx}{2kx + mx} = frac{k}{2k + m} )Third term: ( frac{z}{2z + x} = frac{mx}{2mx + x} = frac{m}{2m + 1} )Therefore, M = ( frac{1}{2 + k} + frac{k}{2k + m} + frac{m}{2m + 1} )Now, we need to maximize this expression with respect to k and m. Hmm, this seems complex, but maybe we can find relationships between k and m. Let me suppose that the maximum occurs when all terms are equal. Let me set each term equal to a constant value, say c. Then:( frac{1}{2 + k} = c )( frac{k}{2k + m} = c )( frac{m}{2m + 1} = c )From the first equation: ( c = frac{1}{2 + k} Rightarrow 2 + k = frac{1}{c} Rightarrow k = frac{1}{c} - 2 )From the third equation: ( c = frac{m}{2m + 1} Rightarrow c(2m +1) = m Rightarrow 2c m + c = m Rightarrow m(1 - 2c) = c Rightarrow m = frac{c}{1 - 2c} )From the second equation: ( c = frac{k}{2k + m} ). Substitute k and m from above.k = (1/c - 2), m = c/(1 - 2c)Thus:( c = frac{1/c - 2}{2*(1/c - 2) + c/(1 - 2c)} )This seems complicated. Let me try to solve for c.Multiply numerator and denominator:Numerator: ( frac{1}{c} - 2 )Denominator: ( 2*(frac{1}{c} - 2) + frac{c}{1 - 2c} )Let me compute denominator:First term: ( 2*(1/c - 2) = 2/c - 4 )Second term: ( c/(1 - 2c) )Therefore, denominator: ( 2/c - 4 + c/(1 - 2c) )So:( c = frac{frac{1}{c} - 2}{frac{2}{c} - 4 + frac{c}{1 - 2c}} )Multiply both sides by denominator:( c*left( frac{2}{c} - 4 + frac{c}{1 - 2c} right) = frac{1}{c} - 2 )Simplify left side:( 2 - 4c + frac{c^2}{1 - 2c} )Right side: ( frac{1}{c} - 2 )So:( 2 - 4c + frac{c^2}{1 - 2c} = frac{1}{c} - 2 )Bring all terms to left side:( 2 - 4c + frac{c^2}{1 - 2c} - frac{1}{c} + 2 = 0 )Combine constants:( 4 - 4c - frac{1}{c} + frac{c^2}{1 - 2c} = 0 )This equation seems quite messy. Let me check if c=1/3 is a solution. Let's see.If c=1/3,Left side:4 - 4*(1/3) - 1/(1/3) + ( (1/3)^2 ) / (1 - 2*(1/3)) )= 4 - 4/3 - 3 + (1/9)/(1 - 2/3)= (4 - 4/3 - 3) + (1/9)/(1/3)= ( (12/3 -4/3 -9/3) ) + (1/9)*(3/1)= (-1/3) + 1/3= 0Yes! So c=1/3 is a solution. Therefore, this suggests that if each term is equal to 1/3, then we have a critical point. So let's check the corresponding k and m.From c=1/3,k = 1/c - 2 = 3 - 2 = 1m = c/(1 - 2c) = (1/3)/(1 - 2/3) = (1/3)/(1/3) =1Therefore, k=1 and m=1, which means y = kx = x and z = mx = x. So x=y=z. Which brings us back to the initial case where all variables are equal, giving M=1. So this suggests that the critical point is indeed at x=y=z.Therefore, the maximum value of M is 1. But wait, earlier when I tested unequal variables, I got lower than 1. However, in some optimization problems, sometimes equal variables give the maximum or minimum. But I need to confirm if this is indeed the maximum.Alternatively, maybe using the Cauchy-Schwarz inequality. Let me think. For each term ( frac{x}{2x + y} ), perhaps I can bound it. Let me consider each term.Note that:( frac{x}{2x + y} = frac{1}{2 + y/x} )Similarly, ( frac{y}{2y + z} = frac{1}{2 + z/y} ), and ( frac{z}{2z + x} = frac{1}{2 + x/z} )Let me set ( a = y/x ), ( b = z/y ), ( c = x/z ). Then, since ( a = y/x ), ( b = z/y ), multiplying these gives ( abc = z/x * x/z = 1 ). So abc=1.Then, M becomes:( frac{1}{2 + a} + frac{1}{2 + b} + frac{1}{2 + c} ), with ( abc =1 ).So now the problem is transformed into maximizing ( frac{1}{2 + a} + frac{1}{2 + b} + frac{1}{2 + c} ) with ( a, b, c >0 ) and ( abc=1 ).This seems like a more symmetric problem. Maybe using Lagrange multipliers here. Let me set function ( f(a,b,c) = frac{1}{2 + a} + frac{1}{2 + b} + frac{1}{2 + c} ), and constraint ( g(a,b,c) = abc -1 =0 ).The Lagrangian is ( mathcal{L} = f(a,b,c) - lambda (abc -1) ).Taking partial derivatives:( frac{partial mathcal{L}}{partial a} = frac{ -1 }{(2 + a)^2} - lambda bc =0 )Similarly,( frac{partial mathcal{L}}{partial b} = frac{ -1 }{(2 + b)^2} - lambda ac =0 )( frac{partial mathcal{L}}{partial c} = frac{ -1 }{(2 + c)^2} - lambda ab =0 )Also, the constraint ( abc =1 ).From the first three equations:( frac{1}{(2 + a)^2} = - lambda bc )( frac{1}{(2 + b)^2} = - lambda ac )( frac{1}{(2 + c)^2} = - lambda ab )Since ( abc=1 ), we can express variables in terms of each other. Let me denote ( a = p ), ( b = q ), ( c = r ), but since they are just variables, let me relate the equations.Divide the first equation by the second:( frac{(2 + b)^2}{(2 + a)^2} = frac{ - lambda bc }{ - lambda ac } = frac{b}{a} )Therefore,( frac{(2 + b)^2}{(2 + a)^2} = frac{b}{a} )Take square roots on both sides:( frac{2 + b}{2 + a} = sqrt{ frac{b}{a} } )Similarly, dividing the second equation by the third:( frac{(2 + c)^2}{(2 + b)^2} = frac{ac}{ab} = frac{c}{b} )Therefore,( frac{(2 + c)^2}{(2 + b)^2} = frac{c}{b} )Take square roots:( frac{2 + c}{2 + b} = sqrt{ frac{c}{b} } )Similarly, dividing the third equation by the first:( frac{(2 + a)^2}{(2 + c)^2} = frac{ab}{bc} = frac{a}{c} )Therefore,( frac{(2 + a)^2}{(2 + c)^2} = frac{a}{c} )Take square roots:( frac{2 + a}{2 + c} = sqrt{ frac{a}{c} } )So, now, we have these three equations:1. ( frac{2 + b}{2 + a} = sqrt{ frac{b}{a} } )2. ( frac{2 + c}{2 + b} = sqrt{ frac{c}{b} } )3. ( frac{2 + a}{2 + c} = sqrt{ frac{a}{c} } )Let me denote equation 1. Let’s set ( t = sqrt{ frac{b}{a} } ). Then ( frac{2 + b}{2 + a} = t ), and ( t = sqrt{ frac{b}{a} } Rightarrow t^2 = frac{b}{a} Rightarrow b = a t^2 ).Substitute into equation 1:( frac{2 + a t^2}{2 + a} = t )Multiply both sides by ( 2 + a ):( 2 + a t^2 = t(2 + a) )Rearrange:( a t^2 - a t - 2 t + 2 =0 )Factor:( a(t^2 - t) -2 t +2 =0 )Similarly, this is a linear equation in a. Solving for a:( a(t^2 - t) = 2 t -2 )So,( a = frac{2 t -2}{t^2 - t} = frac{2(t -1)}{t(t -1)} } = frac{2}{t} )Therefore, ( a = frac{2}{t} )But since ( b = a t^2 ), then ( b = frac{2}{t} * t^2 = 2 t )So we have ( a = 2/t ), ( b = 2 t )Similarly, let's look at equation 2. Let me denote ( s = sqrt{ frac{c}{b} } ). Then, equation 2 becomes ( frac{2 + c}{2 + b} = s ), and ( s = sqrt{ frac{c}{b} } Rightarrow s^2 = c/b Rightarrow c = b s^2 )Substitute into equation 2:( frac{2 + b s^2}{2 + b} = s )Multiply both sides by ( 2 + b ):( 2 + b s^2 = s (2 + b ) )Rearrange:( b s^2 - b s -2 s + 2 =0 )Factor:( b(s^2 - s) -2 s +2 =0 )Solve for b:( b = frac{2 s -2 }{s^2 - s} = frac{2(s -1)}{s(s -1)} } = frac{2}{s} )Therefore, ( b = 2/s )But we already have ( b = 2 t ), so ( 2 t = 2/s Rightarrow t = 1/s )Also, ( c = b s^2 = (2/s) s^2 = 2 s )Similarly, from equation 3, ( frac{2 + a}{2 + c} = sqrt{ frac{a}{c} } )We have a = 2/t = 2 s (since t =1/s), and c =2 s. So,Left side: ( frac{2 + a}{2 + c} = frac{2 + 2 s }{2 + 2 s } =1 )Right side: ( sqrt{ frac{a}{c} } = sqrt{ frac{2 s }{2 s } } =1 )Therefore, equation 3 is satisfied. So, all equations reduce to relations with t and s where t =1/s. Therefore, variables a, b, c can be expressed in terms of t as follows:a = 2/tb = 2 tc = 2 s = 2*(1/t) = 2/tBut wait, abc =1. Let's check:a*b*c = (2/t)*(2 t)*(2/t) = (2/t)*(2 t)*(2/t) = (4)*(2/t) = 8/tBut abc should be 1. Therefore, 8/t =1 ⇒ t=8Therefore, t=8. Then, s=1/t=1/8Therefore,a=2/t=2/8=1/4b=2 t=16c=2 s=2*(1/8)=1/4But abc= (1/4)*16*(1/4)= (1/4)*(4)=1, which satisfies the constraint.Therefore, the critical point is at a=1/4, b=16, c=1/4.Wait, but this seems odd. Let me check what M would be here.M = 1/(2 + a) + 1/(2 + b) +1/(2 + c)= 1/(2 + 1/4) + 1/(2 +16) +1/(2 +1/4)= 1/(9/4) + 1/18 +1/(9/4)= 4/9 + 1/18 +4/9= (4/9 +4/9) +1/18=8/9 +1/18=16/18 +1/18=17/18≈0.944But this is less than 1. So, even though this is a critical point, it's not the maximum. So perhaps the maximum is still at a=1, b=1, c=1, leading to M=1. Hmm, but according to the Lagrangian, this is another critical point. But we need to check if this is a maximum.Alternatively, maybe there are multiple critical points, but the one with a=1, b=1, c=1 is the maximum.Wait, but how did we get a=1/4, b=16, c=1/4 as another critical point? Let me verify the computations.From the Lagrangian, we derived that at critical points, variables must satisfy certain relations, leading to a=2/t, b=2t, c=2/s, with t=1/s. Then, using abc=1, we found t=8. Therefore, leading to a=1/4, b=16, c=1/4. But when we plug these into M, we get 17/18≈0.944, which is less than 1.So this critical point is a local minimum? Or saddle point?Alternatively, maybe when we set up the Lagrangian, there are multiple critical points, and we need to check which one is the maximum.Alternatively, perhaps the maximum occurs at the symmetric point a=b=c=1, which gives M=1.But how to confirm this.Alternatively, since variables are positive and the function is continuous, and considering the behavior as variables approach 0 or infinity, the function seems bounded. For instance, each term is less than 1/2 (since denominator is more than twice the numerator). Therefore, total sum is less than 3/2. But when variables are equal, we get 1, which seems plausible as a maximum.Alternatively, perhaps using the inequality:For positive real numbers a and b, ( frac{a}{2a + b} leq frac{1}{3} ). Wait, let's check:If ( frac{a}{2a + b} leq frac{1}{3} ), then cross-multiplying (since denominators positive):3a ≤ 2a + b ⇒ a ≤ b.But in general, variables can be in any ratio, so this inequality would not hold unless a ≤ b. Therefore, maybe not applicable.Alternatively, let's use the Cauchy-Schwarz inequality on each term.Consider that ( 2x + y = x + x + y geq 3 sqrt[3]{x cdot x cdot y} ) by AM-GM. Wait, but then:Wait, AM-GM on three terms: (x + x + y)/3 ≥ cube root(x*x*y). Therefore,2x + y ≥ 3 (x^2 y)^{1/3}Therefore, ( frac{x}{2x + y} leq frac{x}{3 (x^2 y)^{1/3}} = frac{x}{3 x^{2/3} y^{1/3}} = frac{x^{1/3}}{3 y^{1/3}} = frac{1}{3} left( frac{x}{y} right)^{1/3} )Similarly, each term is bounded by ( frac{1}{3} left( frac{x}{y} right)^{1/3} ), ( frac{1}{3} left( frac{y}{z} right)^{1/3} ), ( frac{1}{3} left( frac{z}{x} right)^{1/3} ).Then, summing them up:( M leq frac{1}{3} left( left( frac{x}{y} right)^{1/3} + left( frac{y}{z} right)^{1/3} + left( frac{z}{x} right)^{1/3} right) )But this might not help directly. Alternatively, since ( left( frac{x}{y} right)^{1/3} + left( frac{y}{z} right)^{1/3} + left( frac{z}{x} right)^{1/3} geq 3 ) by AM-GM, but then this would give M ≤1, which is the case when variables are equal. So this suggests that M ≤1, with equality when x=y=z. Therefore, the maximum is 1.Therefore, the maximum value of M is 1, achieved when x=y=z.Final AnswerThe maximum value of ( M ) is boxed{1}.</think>To find the maximum value of ( M = frac{x}{2x + y} + frac{y}{2y + z} + frac{z}{2z + x} ) where ( x, y, z > 0 ), we start by considering the case where ( x = y = z ). If ( x = y = z ), each term becomes:[frac{x}{2x + x} = frac{x}{3x} = frac{1}{3}]Thus, the sum is:[M = 3 times frac{1}{3} = 1]Next, we consider if there could be a higher value by testing other ratios and using calculus. We compute the partial derivatives of ( M ) with respect to ( x ), ( y ), and ( z ) and set them to zero to find critical points. The partial derivatives lead to a system of equations, which are satisfied when ( x = y = z ).We also use substitutions and inequalities to verify the result. By setting ( x = y = z ), we transform the problem into a symmetric form and apply the AM-GM inequality, which confirms that the maximum value of each term is ( frac{1}{3} ) when ( x = y = z ).Finally, using the Cauchy-Schwarz inequality and checking the behavior of the function as variables approach 0 or infinity, we confirm that the maximum value is indeed achieved when ( x = y = z ).Thus, the maximum value of ( M ) is (boxed{1}).]
🔑:To find the maximum value of ( M = frac{x}{2x + y} + frac{y}{2y + z} + frac{z}{2z + x} ) for ( x, y, z > 0 ), we will use the method of inequalities.1. Initial Setup: We need to show that: [ frac{x}{2x + y} + frac{y}{2y + z} + frac{z}{2z + x} leq 1 ]2. Using the AM-GM Inequality: Consider the following application of the AM-GM (Arithmetic Mean-Geometric Mean) inequality: [ frac{x}{2x + y} leq frac{x}{2x} = frac{1}{2} ] Similarly, [ frac{y}{2y + z} leq frac{y}{2y} = frac{1}{2} ] and [ frac{z}{2z + x} leq frac{z}{2z} = frac{1}{2} ]3. Summing the Inequalities: Adding these inequalities, we get: [ frac{x}{2x + y} + frac{y}{2y + z} + frac{z}{2z + x} leq frac{1}{2} + frac{1}{2} + frac{1}{2} = frac{3}{2} ] However, this approach does not directly help us to prove the desired inequality. We need a different approach.4. Using the Titu's Lemma (Cauchy-Schwarz in Engel Form): Titu's Lemma states that for non-negative real numbers (a_1, a_2, ldots, a_n) and (b_1, b_2, ldots, b_n), [ frac{a_1^2}{b_1} + frac{a_2^2}{b_2} + cdots + frac{a_n^2}{b_n} geq frac{(a_1 + a_2 + cdots + a_n)^2}{b_1 + b_2 + cdots + b_n} ] Applying Titu's Lemma to our problem, we set (a_1 = sqrt{x}), (a_2 = sqrt{y}), (a_3 = sqrt{z}), (b_1 = 2x + y), (b_2 = 2y + z), (b_3 = 2z + x): [ frac{x}{2x + y} + frac{y}{2y + z} + frac{z}{2z + x} geq frac{(x + y + z)^2}{2x + y + 2y + z + 2z + x} = frac{(x + y + z)^2}{3(x + y + z)} = frac{x + y + z}{3} ]5. Conclusion: Since ( frac{x + y + z}{3} leq 1 ) for all positive (x, y, z), we have: [ frac{x}{2x + y} + frac{y}{2y + z} + frac{z}{2z + x} leq 1 ] Therefore, the maximum value of ( M ) is (1).The final answer is (boxed{1}).
❓:Given the ellipse dfrac {x^{2}}{2}+ dfrac {y^{2}}{m}=1 has an eccentricity of dfrac {1}{2}, find m.
🧠:[<think>Okay, so I need to find the value of m for the ellipse given by the equation (x²)/2 + (y²)/m = 1, and the eccentricity is 1/2. Hmm, let me recall what I know about ellipses. First, the standard form of an ellipse is (x²)/(a²) + (y²)/(b²) = 1, where a and b are the lengths of the semi-major and semi-minor axes. But which one is which depends on whether the major axis is along the x-axis or the y-axis. If a > b, then the major axis is along the x-axis, and if b > a, then it's along the y-axis. So in the given equation, the denominators under x² and y² are 2 and m respectively. Therefore, comparing to the standard form, a² is 2 and b² is m, or vice versa, depending on which is larger. Since we don't know m yet, we might have to consider both possibilities.Eccentricity e of an ellipse is given by e = c/a, where c is the distance from the center to a focus, and a is the length of the semi-major axis. Also, c² = a² - b² for an ellipse. So depending on whether the major axis is along x or y, the formula for eccentricity will use different values for a and b.Given that the eccentricity here is 1/2, so e = 1/2. So let's consider both cases.Case 1: The major axis is along the x-axis. Then, a² = 2 (since x² is over 2) and b² = m. Then, the formula for eccentricity is e = c/a, where c = sqrt(a² - b²). Therefore,e = sqrt(a² - b²)/a = sqrt(2 - m)/sqrt(2) = (sqrt(2 - m))/sqrt(2) = sqrt((2 - m)/2)We are told that e = 1/2, so:sqrt((2 - m)/2) = 1/2Let me square both sides to eliminate the square root:(2 - m)/2 = (1/2)² = 1/4Multiply both sides by 2:2 - m = 1/2Then, solving for m:m = 2 - 1/2 = 3/2So in this case, m would be 3/2. But wait, we need to check if this makes sense. If the major axis is along the x-axis, then a² = 2 must be greater than b² = m. Therefore, m must be less than 2. Here, m = 3/2 is 1.5, which is less than 2. So that works.Case 2: The major axis is along the y-axis. Then, a² = m (since y² is over m) and b² = 2. Then, the eccentricity is e = c/a, where c = sqrt(a² - b²). Therefore,e = sqrt(m - 2)/sqrt(m) = sqrt((m - 2)/m)Set this equal to 1/2:sqrt((m - 2)/m) = 1/2Square both sides:(m - 2)/m = 1/4Multiply both sides by m:m - 2 = m/4Subtract m/4 from both sides:(3m)/4 - 2 = 0Add 2:(3m)/4 = 2Multiply both sides by 4:3m = 8Divide by 3:m = 8/3 ≈ 2.666...But in this case, since the major axis is along the y-axis, we must have a² = m > b² = 2, so m must be greater than 2. Here, m = 8/3 ≈ 2.666 is indeed greater than 2, so this is also valid.Wait, but the problem states that the ellipse has an eccentricity of 1/2. So there are two possible values for m? But the question says "find m", implying a unique answer. Hmm. Maybe I did something wrong here. Let me check.Wait, the original ellipse equation is x²/2 + y²/m = 1. So in the standard form, the denominators are a² and b². But whether it's major axis along x or y depends on which denominator is larger. So depending on whether 2 > m or m > 2, the major axis is along x or y, respectively.But the problem didn't specify whether the major axis is along x or y. So both possibilities exist, unless there's some other constraint. But the question is to find m given the eccentricity. So maybe both values are possible? But the problem is stated as "find m", so maybe only one is possible? Wait, let me check.Wait, but the eccentricity formula is different in each case. Let's check if both solutions satisfy their respective cases.In Case 1: m = 3/2. Then, since m = 3/2 < 2, the major axis is along x. Then, e = sqrt((2 - m)/2) = sqrt((2 - 1.5)/2) = sqrt(0.5/2) = sqrt(0.25) = 0.5, which is correct. So that works.In Case 2: m = 8/3 ≈ 2.666. Since m > 2, major axis is along y. Then, e = sqrt((m - 2)/m) = sqrt((8/3 - 2)/(8/3)) = sqrt((8/3 - 6/3)/(8/3)) = sqrt((2/3)/(8/3)) = sqrt(2/8) = sqrt(1/4) = 1/2, which is correct. So both solutions are valid.Therefore, the problem might have two solutions. But the question says "find m". Hmm. Maybe the problem assumes that the major axis is along one of the axes, but perhaps from the equation, it's not specified. Wait, in the given equation, x² term is over 2 and y² is over m. Depending on m's value, the major axis can be along x or y. So if the problem is stated without any further information, both solutions are possible.But the problem is presented as having an answer to "find m", so maybe the original problem has a typo, or maybe in the source problem, there was an additional condition? Wait, perhaps the problem is from a source where m is expected to be a particular value. Wait, but given the problem as stated, both solutions are correct. So is there a mistake in my reasoning?Wait, but in the equation of an ellipse, the denominators must be positive. So 2 and m must be positive, so m > 0. Also, in the case where the major axis is along x, then a² = 2 > b² = m, so m must be less than 2. In the other case, m > 2. So depending on m's value, the major axis is in different directions. Since both possibilities are allowed here, and the problem didn't specify, then technically, there are two solutions: m = 3/2 and m = 8/3.But the problem says "find m". Maybe in the original problem, there was a specification of the major axis direction, but in the given problem here, there isn't. So perhaps the answer is both? Wait, but in the problem statement as presented here, it's just the equation and the eccentricity. So unless there's a standard convention, but in general, ellipses can have major axis along either axis depending on the denominator's size.Wait, but perhaps m is supposed to be under y², so maybe the problem assumes that the major axis is along y-axis. But no, the problem doesn't state that. Wait, unless in the original problem, the equation was written as x²/2 + y²/m =1, and depending on m, it's either a horizontal or vertical ellipse. So, given that, and the fact that eccentricity is 1/2, then m can be either 3/2 or 8/3. Therefore, both values are possible.But maybe I misapplied the formulas? Let me check again.In Case 1: major axis along x. Then, a² = 2, b² = m. Then, e = c/a = sqrt(a² - b²)/a = sqrt(2 - m)/sqrt(2). Set equal to 1/2. So sqrt(2 - m)/sqrt(2) = 1/2. Square both sides: (2 - m)/2 = 1/4. So 2 - m = 1/2 => m = 3/2. Correct.Case 2: major axis along y. Then, a² = m, b² = 2. Then, e = sqrt(a² - b²)/a = sqrt(m - 2)/sqrt(m). Set equal to 1/2: sqrt(m - 2)/sqrt(m) = 1/2. Square both sides: (m - 2)/m = 1/4. Multiply both sides by m: m - 2 = m/4. Subtract m/4: (3m)/4 - 2 = 0 => 3m/4 = 2 => m = 8/3. Correct.So both solutions are valid. Therefore, unless there's a restriction on m (like m < 2 or m > 2) in the problem, both answers are possible. However, the problem as stated does not specify, so perhaps the answer is both 3/2 and 8/3. But in the original problem statement, maybe only one is expected. Wait, let me check the problem again.Original problem: "Given the ellipse x²/2 + y²/m =1 has an eccentricity of 1/2, find m."So the problem is translated as written here. The user specifies "put your final answer within boxed{}", which usually expects a single answer, but perhaps in this case, multiple answers. However, in many textbooks, when the major axis isn't specified, sometimes they consider the case where the larger denominator is under the first term, but in reality, it's determined by which denominator is larger.But given that m is a variable here, the equation can represent both scenarios. Hence, there are two possible values for m. So maybe the answer is both 3/2 and 8/3.But let me check online or think if there's any other approach.Alternatively, maybe there's a miscalculation here. Wait, let's check another way.Suppose the ellipse equation is x²/2 + y²/m =1. Let me recall that for an ellipse, the eccentricity is e = sqrt(1 - (b²/a²)) if it's a horizontal ellipse, or e = sqrt(1 - (a²/b²)) if it's a vertical ellipse? Wait, no, the formula is e = c/a, where c = sqrt(a² - b²) for horizontal, or c = sqrt(b² - a²) for vertical? Wait, no. Wait, regardless of orientation, the formula is e = c/a, where a is the semi-major axis, so if it's a horizontal ellipse, a² > b², so c = sqrt(a² - b²). If it's a vertical ellipse, a² < b², but in that case, the major axis is along the y-axis, so then we usually denote the major axis length as 2b, but wait, no, that might be confusing.Wait, perhaps the confusion arises from notation. Let me clarify.Let me use standard notation where for an ellipse, if it's written as x²/a² + y²/b² =1, then if a > b, it's a horizontal ellipse, major axis along x-axis, with semi-major axis length a, semi-minor axis length b. Then, the eccentricity is e = sqrt(1 - (b²/a²)).If it's written as x²/b² + y²/a² =1, with a > b, then it's a vertical ellipse, major axis along y-axis, semi-major axis length a, semi-minor axis length b, eccentricity e = sqrt(1 - (b²/a²)).But in the problem, the ellipse is written as x²/2 + y²/m =1. So depending on whether 2 > m or m > 2, the major axis is along x or y.Therefore, when we compute the eccentricity, we have two cases. Since the problem gives e = 1/2, and we have to solve for m.In Case 1: 2 > m (horizontal ellipse), so a² = 2, b² = m. Then, e = sqrt(1 - (b²/a²)) = sqrt(1 - m/2). Set equal to 1/2:sqrt(1 - m/2) = 1/2Square both sides:1 - m/2 = 1/4Then,m/2 = 1 - 1/4 = 3/4Multiply by 2:m = 3/2Case 2: m > 2 (vertical ellipse), so a² = m, b² = 2. Then, e = sqrt(1 - (b²/a²)) = sqrt(1 - 2/m). Set equal to 1/2:sqrt(1 - 2/m) = 1/2Square both sides:1 - 2/m = 1/4Then,2/m = 1 - 1/4 = 3/4Multiply both sides by m:2 = (3/4)mDivide both sides by 3/4:m = 2/(3/4) = 8/3So same results. Therefore, m can be either 3/2 or 8/3, depending on whether the major axis is horizontal or vertical. Since the problem doesn't specify, both answers are possible. Therefore, the answer should be m = 3/2 or m = 8/3. But the problem says "find m". If the problem expects both answers, then we need to provide both. However, maybe in some contexts, only one is considered, but according to the standard, both are valid.But let me check the problem statement again. It's given as x²/2 + y²/m =1, and eccentricity 1/2. So maybe m can be either 3/2 or 8/3. Therefore, the answer is both. But since the problem is presented in Chinese or another language, sometimes maybe the notation implies the major axis is along x or y. Wait, in Chinese math problems, sometimes they might specify the major axis, but in this case, it's not.Alternatively, maybe the problem assumes that m is under y², so the major axis is along y. But in the equation, the order is x² term first, then y² term. Typically, in standard form, if it's written as x²/a² + y²/b² =1, then if a > b, it's horizontal. If not, it's vertical. But since m is a variable here, unless specified, both possibilities are open. So both answers are valid.But in the initial problem statement, if they mention "the ellipse", implying a unique ellipse, but given m is variable, unless there's a mistake here. Wait, perhaps m must be positive, but both 3/2 and 8/3 are positive, so that's okay. Alternatively, maybe when solving, the quadratic would lead to two solutions. So in conclusion, since both cases satisfy the equation and the eccentricity, both values are acceptable. Therefore, the answer is m = 3/2 or m = 8/3.But the original problem asks to "find m", and the user here is asking to put the final answer in a box. Usually, for multiple answers, we can list both in separate boxes. But according to the problem's instructions, "put your final answer within boxed{}". The user might expect both answers. But maybe in the original problem, only one is correct. Wait, let me check my calculations once more.Case 1: horizontal ellipse, a²=2, b²=m, e=1/2.e = sqrt(1 - (b²/a²)) = sqrt(1 - m/2) = 1/2Squaring: 1 - m/2 = 1/4 => m/2 = 3/4 => m=3/2. Correct.Case 2: vertical ellipse, a²=m, b²=2, e=1/2.e = sqrt(1 - (b²/a²)) = sqrt(1 - 2/m) = 1/2Squaring: 1 - 2/m = 1/4 => 2/m = 3/4 => m=8/3. Correct.Therefore, both solutions are valid. So unless there's a restriction in the problem, both are correct. Since the problem doesn't specify, the answer is m = 3/2 or 8/3.But maybe the problem assumes that the major axis is along the x-axis. But given that, in the equation, the x² term is first, but in standard form, the major axis is determined by the size of the denominator, not the order. So x²/2 + y²/m =1, if m < 2, major axis is x; if m > 2, major axis is y.Therefore, since the problem doesn't specify, both are acceptable. Therefore, the answer is m = 3/2 or m = 8/3.However, when I look up similar problems, sometimes they may only consider one case, but according to strict mathematics, both are correct. Therefore, the problem might expect both answers. However, the user instruction here is to put the final answer within a single box. But if there are two answers, how to present them? Maybe as boxed{dfrac{3}{2}} and boxed{dfrac{8}{3}}. But the user says "put your final answer within boxed{}". Maybe they expect both answers boxed. But given the problem's phrasing, perhaps the answer is two values. Alternatively, maybe I missed something.Wait, let me check the equation again. The ellipse equation is x²/2 + y²/m =1. If m is positive, then denominators must be positive, so m>0. If m were equal to 2, then it would be a circle. But in our cases, m is either 3/2 or 8/3, which are both positive and not equal to 2, so they are valid ellipses. So both are valid.Alternatively, maybe the problem is designed such that m is supposed to be under y², and since the question is written as x²/2 + y²/m, maybe the major axis is along y, but I don't think so. The major axis is determined by the larger denominator, regardless of the variable's letter.Therefore, I think the correct answer is that m can be either 3/2 or 8/3. Therefore, the answer is both. However, since the problem might expect both values, but the user might want them listed. In Chinese textbooks, sometimes they might expect both answers, separated by commas or something. But in the format here, boxed. So maybe the answer is boxed{dfrac{3}{2}} and boxed{dfrac{8}{3}}.Alternatively, since the problem didn't specify the orientation of the ellipse, both solutions are acceptable, so they should both be presented. Therefore, the answer is m = 3/2 or m = 8/3.Final AnswerThe possible values of ( m ) are boxed{dfrac{3}{2}} and boxed{dfrac{8}{3}}.</think>Given the ellipse (dfrac{x^2}{2} + dfrac{y^2}{m} = 1) with an eccentricity of (dfrac{1}{2}), we need to find the value of (m).First, recall the standard form of an ellipse:- If the major axis is along the x-axis, the equation is (dfrac{x^2}{a^2} + dfrac{y^2}{b^2} = 1) where (a > b).- If the major axis is along the y-axis, the equation is (dfrac{x^2}{b^2} + dfrac{y^2}{a^2} = 1) where (a > b).The eccentricity (e) of an ellipse is given by (e = dfrac{c}{a}), where (c = sqrt{a^2 - b^2}). Case 1: Major axis along the x-axisHere, (a^2 = 2) and (b^2 = m). The eccentricity is:[e = sqrt{dfrac{a^2 - b^2}{a^2}} = sqrt{dfrac{2 - m}{2}}]Given (e = dfrac{1}{2}):[sqrt{dfrac{2 - m}{2}} = dfrac{1}{2}]Squaring both sides:[dfrac{2 - m}{2} = dfrac{1}{4}]Solving for (m):[2 - m = dfrac{1}{2} implies m = 2 - dfrac{1}{2} = dfrac{3}{2}] Case 2: Major axis along the y-axisHere, (a^2 = m) and (b^2 = 2). The eccentricity is:[e = sqrt{dfrac{a^2 - b^2}{a^2}} = sqrt{dfrac{m - 2}{m}}]Given (e = dfrac{1}{2}):[sqrt{dfrac{m - 2}{m}} = dfrac{1}{2}]Squaring both sides:[dfrac{m - 2}{m} = dfrac{1}{4}]Solving for (m):[m - 2 = dfrac{m}{4} implies 4m - 8 = m implies 3m = 8 implies m = dfrac{8}{3}] ConclusionBoth values of (m) are valid depending on whether the major axis is along the x-axis or y-axis. Therefore, the possible values of (m) are:[boxed{dfrac{3}{2}} text{ and } boxed{dfrac{8}{3}}],<think>Okay, so I have this problem here: Given the ellipse (x²/2) + (y²/m) = 1, and it has an eccentricity of 1/2. I need to find m. Alright, let's start by recalling what I know about ellipses. First, the standard form of an ellipse is (x²/a²) + (y²/b²) = 1, where a and b are the lengths of the semi-major and semi-minor axes. But which one is major and which is minor depends on whether the ellipse is oriented horizontally or vertically. If a² > b², then the major axis is along the x-axis, and if b² > a², then it's along the y-axis. The eccentricity e is given by e = c/a, where c is the distance from the center to a focus, and a is the length of the semi-major axis. Also, c² = a² - b² for an ellipse. So, the formula for eccentricity can also be written as e = √(1 - (b²/a²)) if it's a horizontal ellipse, or e = √(1 - (a²/b²)) if it's vertical. Hmm, I need to be careful here.Given the equation (x²/2) + (y²/m) = 1, I need to figure out whether it's a horizontal or vertical ellipse. To do that, I should compare the denominators under x² and y². The denominator under x² is 2, and under y² is m. So, if 2 > m, then the major axis is along the x-axis, and the ellipse is horizontal. If m > 2, then the major axis is along the y-axis, making it a vertical ellipse. The problem doesn't specify, so m could be either greater than 2 or less than 2. But depending on the value of m, the ellipse orientation changes. Since the eccentricity is given as 1/2, I need to consider both possibilities and see which one gives a valid m.Let me write down the formulas for both cases.Case 1: Horizontal ellipse (a² = 2, b² = m). Then, the eccentricity e = √(1 - (b²/a²)) = √(1 - (m/2)). Given that e = 1/2, set up the equation: √(1 - (m/2)) = 1/2. Squaring both sides: 1 - (m/2) = 1/4. Then, solving for m: m/2 = 1 - 1/4 = 3/4, so m = (3/4)*2 = 3/2. So m = 3/2. But wait, this is under the assumption that the ellipse is horizontal, which requires that a² > b², which would mean 2 > m. So m = 3/2 is 1.5, which is less than 2. So that's valid. So in this case, m = 3/2.Case 2: Vertical ellipse (b² = 2, a² = m). Then, the eccentricity e = √(1 - (b²/a²)) = √(1 - (2/m)). Given that e = 1/2, set up the equation: √(1 - (2/m)) = 1/2. Squaring both sides: 1 - (2/m) = 1/4. Then, 2/m = 1 - 1/4 = 3/4. So, 2/m = 3/4, solving for m: m = 2*(4/3) = 8/3 ≈ 2.666... But here, since it's a vertical ellipse, we need a² > b², meaning m > 2. 8/3 is approximately 2.666, which is greater than 2, so that's valid. So m could also be 8/3.Wait a minute, so there are two possible values of m? Depending on whether the major axis is horizontal or vertical? But the problem doesn't specify. Hmm, but maybe I need to check whether both are possible. Let me verify.First, in the original equation, (x²/2) + (y²/m) = 1. For this to be an ellipse, both denominators must be positive. So 2 > 0 and m > 0. So m must be positive, but that's already given. Since the problem states it's an ellipse, so m can't be equal to 2, because then we would have a circle if m=2, but since m is different from 2, it's an ellipse. But in our case, we have two possible solutions: m = 3/2 or 8/3. So both are valid? But the problem is asking to find m, so maybe both answers are possible? Wait, the problem is phrased as "find m", which might suggest there's a unique answer. Maybe I need to check if both solutions are acceptable or if one of them is invalid.Wait, but both cases lead to valid ellipses with eccentricity 1/2, just oriented differently. So perhaps both values of m are acceptable. But maybe the problem expects both answers? Let me check again.Wait, but maybe I made a mistake in the formulas. Let me go through each case again carefully.Case 1: Horizontal ellipse. Here, a² = 2 (since under x²), b² = m. Then, the formula for eccentricity is e = c/a, where c = sqrt(a² - b²). So, e = sqrt(1 - b²/a²). So, e = sqrt(1 - m/2). Set that equal to 1/2. Then,sqrt(1 - m/2) = 1/2Square both sides:1 - m/2 = 1/4Then,m/2 = 1 - 1/4 = 3/4Multiply both sides by 2:m = 3/2Which is 1.5, less than 2, so that's a valid horizontal ellipse.Case 2: Vertical ellipse. Here, a² = m (under y²), b² = 2 (under x²). Then, e = sqrt(1 - b²/a²) = sqrt(1 - 2/m). Set equal to 1/2:sqrt(1 - 2/m) = 1/2Square both sides:1 - 2/m = 1/4Then,2/m = 1 - 1/4 = 3/4Multiply both sides by m:2 = (3/4)mThen,m = 2 * (4/3) = 8/3 ≈ 2.666...Which is greater than 2, so that's a valid vertical ellipse.Therefore, both values of m are possible. But the problem says "the ellipse has an eccentricity of 1/2", and the equation is given as (x²/2) + (y²/m) = 1. Since m could be either 3/2 or 8/3, depending on orientation, but the problem doesn't specify the orientation, so maybe both are acceptable. But let me check the original problem again.The problem says: "Given the ellipse (x²)/2 + (y²)/m =1 has an eccentricity of 1/2, find m."Wait, but in the original problem statement, maybe they are following a convention where the major axis is along the x-axis if a² is under x², but only if a² > b². But since in the equation, the denominators are 2 and m, which is variable. So, perhaps both possibilities must be considered. Therefore, m can take two values: 3/2 and 8/3. But the problem might be expecting both? But the problem says "find m", which might imply a single answer, so perhaps I need to check if both are valid.Wait, but in the problem statement, when they write the equation as (x²)/2 + (y²)/m =1, depending on m's value, the major axis is x or y. Since m is a variable here, maybe both solutions are possible. Therefore, the answer is m = 3/2 or m = 8/3.But maybe the problem is expecting both answers. Wait, let me check the question again. It says "find m". So perhaps both values are acceptable, and we need to present both. Let me check if my calculations are correct.First case: horizontal ellipse, a²=2, e=1/2. Then, e = sqrt(1 - (m/2)) => 1/2 = sqrt(1 - m/2). Squared: 1/4 = 1 - m/2 => m/2 = 3/4 => m= 3/2. Correct.Second case: vertical ellipse, a²=m, e=1/2. Then, e = sqrt(1 - (2/m)) => 1/2 = sqrt(1 - 2/m). Squared: 1/4 = 1 - 2/m => 2/m = 3/4 => m=8/3. Correct.So both solutions are valid, hence m could be either 3/2 or 8/3.But let me check once more: if m=3/2, then the equation is (x²)/2 + (y²)/(3/2) =1, which simplifies to (x²)/2 + (2y²)/3 =1. The semi-major axis is sqrt(2) along x, semi-minor is sqrt(3/2). Then, the eccentricity should be sqrt(1 - (b²/a²)) = sqrt(1 - ( (3/2)/2 )) = sqrt(1 - 3/4) = sqrt(1/4) = 1/2. Correct.For m=8/3, the equation is (x²)/2 + (y²)/(8/3) =1, which is (x²)/2 + (3y²)/8 =1. Here, semi-major axis is sqrt(8/3) along y, semi-minor is sqrt(2). Then, eccentricity is sqrt(1 - (b²/a²)) = sqrt(1 - (2/(8/3))) = sqrt(1 - (6/8)) = sqrt(1 - 3/4) = sqrt(1/4)=1/2. Correct.Therefore, both values of m are valid. So the answer is m=3/2 or m=8/3. However, the problem might have intended only one solution. Wait, maybe the problem assumes the ellipse is horizontal or vertical? Let me check the original problem statement again.It just says "the ellipse (x²)/2 + (y²)/m =1 has an eccentricity of 1/2, find m". There's no mention of major axis direction, so both possibilities must be considered, leading to two solutions. Therefore, the answer should be m=3/2 and m=8/3.But I need to check if the problem allows multiple answers. The original problem didn't specify whether m is greater than 2 or less than 2, so both are possible. Therefore, m can be either 3/2 or 8/3. So, unless there's a mistake in my calculations, both are correct. But let me check once again.Wait, perhaps I confused a and b in the vertical case. Let me re-express the standard forms. For a vertical ellipse, the standard form is (x²/b²) + (y²/a²) =1, where a > b. Then, the formula for eccentricity is e = sqrt(1 - (b²/a²)). So in this case, if the given ellipse is vertical, then the denominator under y² is a², which is m, so m =a², and the denominator under x² is b²=2. Therefore, e = sqrt(1 - (b²/a²))=sqrt(1 - 2/m). Setting equal to 1/2, solving gives m=8/3. Correct.Similarly, if the ellipse is horizontal, then a²=2 and b²=m, so e = sqrt(1 - m/2) =1/2. Solving gives m=3/2. Correct. Therefore, both are valid. Therefore, the problem has two solutions.But the original problem says "find m". Maybe the answer is both. However, in some textbooks, they might assume the major axis is along the larger denominator, so if it's written as x²/2 + y²/m =1, then if m >2, it's vertical, if m <2, it's horizontal. But since m is a variable here, both cases are possible. Therefore, the answer is m= 3/2 or 8/3.But in the problem statement, they might want both values. However, the user hasn't specified multiple answers, so maybe the answer is both? Let me see.Wait, perhaps the problem is from a source where multiple answers are possible, but in the initial problem, maybe the user expects only one answer. But according to my calculations, there are two possible answers. Therefore, the answer is m=3/2 and m=8/3.But to confirm, let me check an example. Suppose m=3/2, then the ellipse is (x²)/2 + (y²)/(3/2) =1. Let's compute the eccentricity: a²=2, b²=3/2. Then, c² =a² -b²=2 - 3/2=1/2. So, c=√(1/2)=√2/2. Then, e= c/a= (√2/2)/√2= (√2/2)*(1/√2)=1/2. Correct.Similarly, for m=8/3: (x²)/2 + (y²)/(8/3)=1. Here, a²=8/3, b²=2. Then, c²=a² -b²=8/3 -2=8/3 -6/3=2/3. So c=√(2/3). Then, e= c/a= √(2/3)/√(8/3)= (√(2)/√(3)) / (2√2/√3))= (√2 / √3) * (√3 / 2√2)=1/2. Correct. Therefore, both are valid.Therefore, the answer is m=3/2 or m=8/3. But the problem may require both answers. Let me check if the original problem is from a source that expects multiple answers. However, since the problem is presented as "find m", which is in the title, maybe the answer is both. But perhaps the original problem assumes that m >2 or m <2. Wait, but unless specified, both are possible. Therefore, the answer is both 3/2 and 8/3. So I should present both values. However, sometimes problems may expect the answer in a specific form, but since the user hasn't given more context, we need to assume both are correct. Alternatively, maybe I made a mistake here. Let me check once more.Wait, the formula for eccentricity for an ellipse is always e = c/a, where a is the semi-major axis. So, regardless of the orientation, you take a as the larger one. So in the equation (x²)/2 + (y²)/m =1, if m >2, then the semi-major axis is sqrt(m), and the semi-minor is sqrt(2). If m <2, then semi-major is sqrt(2), semi-minor is sqrt(m). Therefore, depending on m's relation to 2, we have two cases. Since the eccentricity is given as 1/2, which is less than 1, so both cases are possible. Therefore, m can be either 3/2 or 8/3. Therefore, the answer is both. So, the problem is designed to have two solutions. Therefore, the answer is m=3/2 or m=8/3.Thus, I should box both answers. But in the initial problem, how is the answer expected? Since the user instruction says "put your final answer within boxed{}", and if there are multiple answers, maybe we need to present both boxed. But I need to check if the system allows multiple boxes. Alternatively, maybe the problem expects only one answer, but according to the calculations, both are valid. Therefore, I need to state both. Therefore, the answer is m = boxed{dfrac{3}{2}} and m = boxed{dfrac{8}{3}}.But let me check if the problem could have a unique answer. Suppose m is meant to be under y², so depending on the value of m, but the problem might have a unique solution. Wait, no, since both cases give valid eccentricity. So, both answers are correct.Alternatively, maybe I need to check if m=2 is a possibility, but m=2 would make the ellipse a circle, but the eccentricity of a circle is 0, not 1/2, so m=2 is invalid. So, only 3/2 and 8/3 are possible. Therefore, the answer is both.Therefore, the problem has two solutions, m=3/2 and m=8/3.Final AnswerThe possible values of ( m ) are boxed{dfrac{3}{2}} and boxed{dfrac{8}{3}}.</think>Given the ellipse (dfrac{x^2}{2} + dfrac{y^2}{m} = 1) with an eccentricity of (dfrac{1}{2}), we need to find the value of (m).First, recall the standard form of an ellipse (dfrac{x^2}{a^2} + dfrac{y^2}{b^2} = 1) where (a) and (b) are the lengths of the semi-major and semi-minor axes. The eccentricity (e) is given by (e = dfrac{c}{a}) where (c = sqrt{a^2 - b^2}). The eccentricity can also be expressed as (e = sqrt{1 - dfrac{b^2}{a^2}}) for a horizontal ellipse and (e = sqrt{1 - dfrac{a^2}{b^2}}) for a vertical ellipse.For the given equation (dfrac{x^2}{2} + dfrac{y^2}{m} = 1), we consider two cases based on whether the ellipse is horizontal or vertical:Case 1: Horizontal ellipse ((a^2 = 2) and (b^2 = m))- The eccentricity is (e = sqrt{1 - dfrac{m}{2}}).- Setting (e = dfrac{1}{2}), we solve: [ sqrt{1 - dfrac{m}{2}} = dfrac{1}{2} ] Squaring both sides: [ 1 - dfrac{m}{2} = dfrac{1}{4} ] Solving for (m): [ dfrac{m}{2} = dfrac{3}{4} implies m = dfrac{3}{2} ]Case 2: Vertical ellipse ((a^2 = m) and (b^2 = 2))- The eccentricity is (e = sqrt{1 - dfrac{2}{m}}).- Setting (e = dfrac{1}{2}), we solve: [ sqrt{1 - dfrac{2}{m}} = dfrac{1}{2} ] Squaring both sides: [ 1 - dfrac{2}{m} = dfrac{1}{4} ] Solving for (m): [ dfrac{2}{m} = dfrac{3}{4} implies m = dfrac{8}{3} ]Both values of (m) are valid as they correspond to horizontal and vertical ellipses respectively. Therefore, the possible values of (m) are:[boxed{dfrac{3}{2}} text{ and } boxed{dfrac{8}{3}}]]
🔑:From the standard equation of the ellipse, we have:(1) When 0 2, we get b= sqrt {2}, a= sqrt {m},thus c= sqrt {-2+m}, so the eccentricity of the ellipse e= dfrac {c}{a}= dfrac { sqrt {-2+m}}{ sqrt {2}}= dfrac {1}{2}.We find m= dfrac {8}{3};In summary, m= dfrac {3}{2} or dfrac {8}{3}Therefore, the answer is: boxed{dfrac {3}{2} text{ or } dfrac {8}{3}}By using the standard equation of the ellipse to find the values of a and b, and then using a^{2}=b^{2}+c^{2} to find the value of c, and applying the formula for eccentricity e= dfrac {c}{a}, we can establish an equation in terms of m to find its value. This question tests the student's understanding of how to find the eccentricity of an ellipse, the flexible application of the ellipse's simple properties to simplify and solve for values, and emphasizes the need for case discussion. It is a basic question.
❓:(1) Calculate: left(-2right)^{2}times sqrt{frac{1}{4}}+|sqrt[3]{-8}|+sqrt{2}times left(-1right)^{2023};(2) Given that the square root of 2a-1 is pm 3, and the cube root of 11a+b-1 is 4, find the square root of a+2b.
🧠:[<think>Okay, let's tackle these two problems step by step. I need to make sure I understand each part and don't make any mistakes. Let me start with problem (1).Problem (1): Calculate: ((-2)^{2} times sqrt{frac{1}{4}} + |sqrt[3]{-8}| + sqrt{2} times (-1)^{2023}).Alright, so this expression has several parts. Let me break it down term by term.First term: ((-2)^{2}). Hmm, okay, exponents come before multiplication, right? So, (-2) squared. Since the exponent is even, the result should be positive. Let me calculate that: (-2 times -2 = 4). Got that, so the first term is 4.Second term: (sqrt{frac{1}{4}}). The square root of 1/4. Well, the square root of 1 is 1, and the square root of 4 is 2. So, (sqrt{frac{1}{4}} = frac{1}{2}). That seems straightforward.So the first part of the expression is (4 times frac{1}{2}). Let me compute that: 4 times 1/2 is 2. Okay, so that part is 2.Next term: (|sqrt[3]{-8}|). The cube root of -8. The cube root of -8 is -2 because (-2)^3 is -8. Then, taking the absolute value of that: |-2| is 2. So that term is 2. Got that.Third term: (sqrt{2} times (-1)^{2023}). Let's see. (-1) raised to an odd exponent is -1, right? Since 2023 is odd. So, (-1)^2023 = -1. Then, multiplying that by sqrt(2): sqrt(2) * (-1) = -sqrt(2). So that term is -sqrt(2).Now, putting all those together: first part was 2, second term is 2, third term is -sqrt(2). So adding them up: 2 + 2 - sqrt(2). That simplifies to 4 - sqrt(2).Wait, let me verify each step again to make sure I didn't skip anything.First term: (-2)^2 = 4. Correct. Then multiplied by sqrt(1/4) = 1/2. So 4 * 1/2 = 2. Correct.Second term: cube root of -8 is -2, absolute value is 2. Correct.Third term: sqrt(2) * (-1)^2023. Since 2023 is odd, (-1)^2023 = -1. So sqrt(2) * (-1) = -sqrt(2). Correct.Adding all: 2 (from first term) + 2 (from second term) + (-sqrt(2)) (from third term). So 2 + 2 = 4, then 4 - sqrt(2). That's right.So the answer to problem (1) is 4 - sqrt(2). Let me note that down.Now moving on to problem (2):Problem (2): Given that the square root of (2a - 1) is (pm 3), and the cube root of (11a + b - 1) is 4, find the square root of (a + 2b).Alright, let's parse this. The square root of (2a - 1) is (pm 3). Hmm, when they say the square root is (pm 3), that means that (2a - 1) is equal to the square of 3, right? Because if sqrt(x) = ±k, then x = k². So here, sqrt(2a - 1) = ±3 implies that 2a - 1 = (3)² = 9. Let me confirm that.Yes, if we have sqrt(x) = ±k, then squaring both sides gives x = k². So 2a - 1 = 9. Solving for a: 2a = 9 + 1 = 10, so a = 10 / 2 = 5. So a is 5. Got that.Next part: the cube root of (11a + b - 1) is 4. Cube root, so if cube root(y) = 4, then y = 4³ = 64. So, 11a + b - 1 = 64.We already found that a = 5, so plug that into the equation: 11*5 + b - 1 = 64. Let's compute 11*5: 55. So 55 + b - 1 = 64. That simplifies to 54 + b = 64. Therefore, b = 64 - 54 = 10. So b is 10.Now, we need to find the square root of (a + 2b). Let's compute (a + 2b). Since a = 5 and b = 10, that's 5 + 2*10 = 5 + 20 = 25. So, the square root of 25 is ±5.Wait, but hold on. The problem says "find the square root of (a + 2b)". So since (a + 2b = 25), its square roots are ±5. Therefore, the answer is ±5.Let me check again to make sure I didn't make any mistakes.First equation: sqrt(2a -1) = ±3 => 2a -1 = 9 => 2a =10 => a=5. Correct.Second equation: cube root(11a + b -1) =4 => 11a +b -1 =64. Substituting a=5: 55 + b -1 =64 => 54 + b=64 => b=10. Correct.Then a + 2b=5 + 20=25. Square root of 25 is ±5. Yes, that seems right.So the square root of a + 2b is ±5.So summarizing:Problem (1) answer is 4 - sqrt(2).Problem (2) answer is ±5.I need to make sure there are no calculation errors here. Let me check problem (1) once more.Original expression:(-2)^2 * sqrt(1/4) + |cbrt(-8)| + sqrt(2)*(-1)^2023Calculations:(-2)^2 =4. sqrt(1/4)=1/2. 4*(1/2)=2.cbrt(-8)= -2. | -2 | =2.sqrt(2)*(-1)^2023: 2023 is odd, so (-1)^2023= -1. So sqrt(2)*(-1)= -sqrt(2).Adding all terms: 2 +2 +(-sqrt(2))=4 - sqrt(2). Correct.Problem (2):From sqrt(2a -1)=±3 => 2a -1=9 => a=5.From cbrt(11a + b -1)=4 =>11a +b -1=64. Substitute a=5: 55 +b -1=64 =>b=10.Then a +2b=5 +20=25. sqrt(25)=±5. Correct.Everything checks out. So the answers should be as above.Final Answer(1) boxed{4 - sqrt{2}} (2) boxed{pm 5}</think>(1) Calculate: ((-2)^{2} times sqrt{frac{1}{4}} + |sqrt[3]{-8}| + sqrt{2} times (-1)^{2023}).- First term: ((-2)^2 = 4)- Second term: (sqrt{frac{1}{4}} = frac{1}{2})- Multiplying the first two terms: (4 times frac{1}{2} = 2)- Third term: (|sqrt[3]{-8}| = |-2| = 2)- Fourth term: (sqrt{2} times (-1)^{2023} = sqrt{2} times (-1) = -sqrt{2})Adding all terms together: (2 + 2 - sqrt{2} = 4 - sqrt{2})(2) Given that the square root of (2a - 1) is (pm 3), and the cube root of (11a + b - 1) is 4, find the square root of (a + 2b).- From (sqrt{2a - 1} = pm 3), we get (2a - 1 = 9). Solving for (a): (2a = 10 implies a = 5).- From (sqrt[3]{11a + b - 1} = 4), we get (11a + b - 1 = 64). Substituting (a = 5): (11 times 5 + b - 1 = 64 implies 55 + b - 1 = 64 implies b = 10).- Calculating (a + 2b): (5 + 2 times 10 = 25). The square root of 25 is (pm 5).Final Answers:(1) boxed{4 - sqrt{2}} (2) boxed{pm 5},<think>Alright, let's tackle these problems one by one. Starting with part (1). Hmm, okay, so we have to calculate this expression: (-2)² times the square root of 1/4, plus the absolute value of the cube root of -8, plus the square root of 2 times (-1) raised to the 2023rd power. Let me break this down step by step.First, let's handle each term separately. The first term is (-2) squared. Remember, when you square a negative number, the result is positive. So (-2)² is 4. Got that.Next, we multiply that by the square root of 1/4. The square root of 1/4 is... well, what's the square of 1/2? It's 1/4. So sqrt(1/4) is 1/2. So 4 times 1/2 is... 2. Okay, so the first part simplifies to 2.Moving on to the second term: the absolute value of the cube root of -8. The cube root of -8 is -2, because (-2)³ is -8. Then, taking the absolute value of that, |-2| is 2. So the second term is 2.Third term: sqrt(2) multiplied by (-1) raised to the 2023rd power. Let's see, (-1) raised to an odd exponent is -1. Since 2023 is odd, (-1)^2023 is -1. So this term is sqrt(2) * (-1) which is -sqrt(2). Now, adding up all the terms: 2 (from the first part) + 2 (from the second part) + (-sqrt(2)). So that's 2 + 2 - sqrt(2). 2 + 2 is 4, so the entire expression simplifies to 4 - sqrt(2). Wait, let me double-check each step to make sure I didn't make a mistake. First term: (-2)² is indeed 4. Square root of 1/4 is 1/2. Multiplying 4 and 1/2 gives 2. Correct. Second term: cube root of -8 is -2, absolute value makes it 2. Third term: sqrt(2) times (-1)^2023. Since 2023 is odd, yes, that's -1. So sqrt(2)*(-1) is -sqrt(2). Adding them together: 2 + 2 = 4; 4 - sqrt(2). That seems right. I don't see any errors here. Okay, so part (1) is 4 - sqrt(2). Moving on to part (2). The problem states: the square root of (2a - 1) is ±3, and the cube root of (11a + b -1) is 4. Find the square root of (a + 2b). Let me parse this. If the square root of (2a -1) is ±3, that means that 2a -1 is equal to (3)^2 or (-3)^2? Wait, if sqrt(2a -1) is ±3, that would mean that 2a -1 is equal to 9, because sqrt(x) = ±k implies x = k². Because sqrt(x) is the principal root, but if they say the square root is ±3, that's a bit confusing. Wait, actually, perhaps the equation is sqrt(2a -1) = ±3. But in reality, the square root function only outputs non-negative results. So maybe the original equation is that 2a -1 squared is 9? Wait, no. Let me think. Wait, when they say "the square root of 2a -1 is ±3", that might be a way of saying that 2a -1 is a number whose square roots are ±3. But the square roots of a number can't be both positive and negative unless the number is positive. Wait, if the square root of something is ±3, then that something must be 9. Because both 3 and -3 squared are 9. So sqrt(9) is 3, but if they're considering both roots, then it's ±3. So perhaps the equation is 2a -1 = 9. Because if sqrt(2a -1) = ±3, then squaring both sides gives 2a -1 = 9. So that's the way to interpret that. Similarly, the cube root of (11a + b -1) is 4. So cube root of something equals 4, so that something must be 4 cubed, which is 64. So 11a + b -1 = 64. So from the first equation: 2a -1 = 9. Solving for a: add 1 to both sides, 2a = 10, so a = 5. Then, substitute a = 5 into the second equation: 11(5) + b -1 = 64. Let's compute 11*5: that's 55. So 55 + b -1 = 64. So 54 + b = 64. Subtract 54: b = 64 - 54 = 10. So a = 5, b = 10. Then, we need to find the square root of (a + 2b). Let's compute a + 2b: 5 + 2*10 = 5 + 20 = 25. The square root of 25 is 5. Therefore, the square root of a + 2b is 5. Wait, let me verify each step again. First equation: sqrt(2a -1) = ±3. But sqrt(2a -1) is a real number only if 2a -1 >= 0, which would be a >= 0.5. But since sqrt(2a -1) is given as ±3, but the square root function itself only gives the principal (non-negative) root. So if they say that the square root is ±3, that might be a bit of a misstatement. However, in algebra, when we say "the square root of x is ±k", we usually mean that x = k². So 2a -1 = (±3)² = 9. Therefore, 2a -1 = 9, leading to a = 5. That seems correct. Second equation: cube root of (11a + b -1) = 4. Therefore, 11a + b -1 = 4³ = 64. Plugging a =5: 11*5 = 55. So 55 + b -1 = 64. 55 -1 is 54, so 54 + b = 64. Therefore, b = 10. That's correct. Then a + 2b = 5 + 2*10 = 25. Square root of 25 is 5. So the answer is 5. Makes sense. Wait, but the question says "the square root of a + 2b". So sqrt(a + 2b) is sqrt(25) = 5. However, technically, the square root of 25 is 5 (principal root), but depending on how it's phrased, could it be ±5? The problem says "find the square root of...", so if they follow the principal root, it's 5, but sometimes problems might expect both roots. But given that in part (1), they mentioned the square root with ±, but here they just say "the square root". Hmm. But in the first part, they wrote sqrt(1/4) which is 1/2, so maybe they consider the principal root. Also, when they mentioned "the square root of 2a -1 is ±3", they used the ±, so perhaps when they want both roots, they specify. Here, since the problem just says "find the square root", it's likely expecting the principal (non-negative) root. So the answer is 5. Therefore, part (2) answer is 5. So, compiling my answers: part (1) is 4 - sqrt(2), and part (2) is 5. Let me just verify once again for any possible miscalculations. For part (1): (-2)^2 is 4, sqrt(1/4) is 1/2, so 4 * 1/2 = 2. Cube root of -8 is -2, absolute value is 2. Then sqrt(2) * (-1)^2023: since 2023 is odd, (-1)^2023 is -1, so sqrt(2)*(-1) is -sqrt(2). Adding up: 2 + 2 - sqrt(2) = 4 - sqrt(2). Correct. For part (2): 2a -1 = 9 => a=5, then 11a + b -1 = 64 => 55 + b -1 = 64 => b=10. Then a + 2b = 5 + 20 =25, sqrt(25)=5. Correct. I think that's all. No mistakes detected.Final Answer(1) boxed{4 - sqrt{2}}; (2) boxed{5}</think>(1) Calculate: ((-2)^{2} times sqrt{frac{1}{4}} + |sqrt[3]{-8}| + sqrt{2} times (-1)^{2023}).- First term: ((-2)^2 = 4).- Second term: (sqrt{frac{1}{4}} = frac{1}{2}). Multiplying by 4 gives (4 times frac{1}{2} = 2).- Third term: (sqrt[3]{-8} = -2). Taking the absolute value gives (|-2| = 2).- Fourth term: ((-1)^{2023} = -1). Multiplying by (sqrt{2}) gives (sqrt{2} times (-1) = -sqrt{2}).Adding all terms together: (2 + 2 - sqrt{2} = 4 - sqrt{2}).(2) Given that the square root of (2a - 1) is (pm 3), and the cube root of (11a + b - 1) is 4, find the square root of (a + 2b).- From the first equation: (sqrt{2a - 1} = pm 3) implies (2a - 1 = 9). Solving for (a) gives (2a = 10) so (a = 5).- From the second equation: (sqrt[3]{11a + b - 1} = 4) implies (11a + b - 1 = 64). Substituting (a = 5) gives (55 + b - 1 = 64). Solving for (b) gives (54 + b = 64) so (b = 10).Calculating (a + 2b): (5 + 2 times 10 = 5 + 20 = 25). The square root of 25 is 5.Final answers:(1) (boxed{4 - sqrt{2}});(2) (boxed{5})]
🔑: Problem 1 Calculation:Let's break down the calculation step by step:1. Calculate left(-2right)^{2}:left(-2right)^{2} = 42. Calculate sqrt{frac{1}{4}}:sqrt{frac{1}{4}} = frac{1}{2}3. Calculate |sqrt[3]{-8}|:|sqrt[3]{-8}| = |{-2}| = 24. Calculate sqrt{2}times left(-1right)^{2023}:Since 2023 is odd, left(-1right)^{2023} = -1, thus:sqrt{2}times left(-1right)^{2023} = sqrt{2}times (-1) = -sqrt{2}Combining all parts:left(-2right)^{2}times sqrt{frac{1}{4}}+|sqrt[3]{-8}|+sqrt{2}times left(-1right)^{2023} = 4times frac{1}{2}+2-sqrt{2} = 2+2-sqrt{2} = 4-sqrt{2}Therefore, the final answer is:boxed{4-sqrt{2}} Problem 2 Solution:Given:1. The square root of 2a-1 is pm 3.2. The cube root of 11a+b-1 is 4.Let's solve these step by step:1. From the square root of 2a-1 being pm 3, we have:2a-1 = 9Solving for a:2a = 10 Rightarrow a = 52. From the cube root of 11a+b-1 being 4, substituting a = 5:11a+b-1 = 6411(5)+b-1 = 64Solving for b:55+b-1 = 64 Rightarrow b = 10Now, calculate a+2b:a+2b = 5+2times 10 = 25Therefore, the square root of a+2b is:sqrt{25} = pm 5Thus, the final answer is:boxed{pm 5}
❓:2. Consider an n times n table, with n geqslant 1. Aya wants to color k cells of this table so that there is a unique way to place n tokens on colored cells without any two tokens being in the same row or the same column. What is the maximum value of k for which Aya's wish is achievable?## Solutions
🧠:[<think>Okay, let's try to figure out this problem. So, we have an n x n table, and Aya wants to color k cells such that there's a unique way to place n tokens on the colored cells without any two tokens being in the same row or column. We need to find the maximum k where this is possible. Hmm, interesting.First, let me make sure I understand the problem correctly. The key points are:1. We color k cells in an n x n grid.2. After coloring, there must be exactly one way to place n tokens such that each token is in a unique row and column (i.e., a permutation matrix, which is a Latin square with exactly one non-zero entry in each row and column).3. We need the maximum k where this uniqueness is achievable.So, the question is about the maximum number of colored cells that still allows only one perfect matching (in the bipartite graph sense, where rows and columns are nodes, and colored cells are edges). The maximum k such that the bipartite graph has a unique perfect matching.I remember that in bipartite graphs, the number of perfect matchings can be related to the structure of the graph. For a graph to have a unique perfect matching, it must be constrained in some way.First, think about small n. Maybe starting with n=1. Then, the grid is 1x1. Coloring the only cell gives k=1, and there's trivially a unique placement. So maximum k is 1 here.For n=2. Let's see. If we color all 4 cells, then there are 2 possible permutations: (1,2) and (2,1). So if k=4, there are two perfect matchings. So that's too much. If we color 3 cells, maybe. Let's say we color three cells. Then, depending on which three, there could still be two perfect matchings. For example, if we leave out one cell, but the remaining three form a sort of L-shape with one cell missing. Wait, in a 2x2 grid, if you color three cells, the only way to have a unique perfect matching is if the missing cell is in such a way that only one permutation is possible. Let me draw it.Suppose the grid is:a bc dIf we color a, b, c, then the colored cells are a, b, c. The possible permutations are a and c (if you pick a from the first row, then you have to pick c from the second row; if you pick b from the first row, there's no cell in the second row's second column). Wait, but the second row only has c colored. So if you pick b in the first row, then in the second row, you have to pick c, but c is in column 1, which is already taken by a. Wait, no. Let's see. In the first row, you can pick a or b. If you pick a, then in the second row, you can only pick c (since column 1 is already used). If you pick b in the first row, then in the second row, you can only pick c again (since column 2 is used). But c is in column 1, so if you pick c, column 1 is used. Wait, but if you pick b in the first row (column 2), then in the second row, the only colored cell is c (column 1). So that's allowed. So actually, there are two perfect matchings: (a, c) and (b, c). Wait, but in the second case, is it (b, c)? That would mean two tokens in column 1 and 2. Wait, no. If you pick b (row 1, column 2), then in row 2, the only available cell is c (row 2, column 1). So yes, that is a valid permutation: (1,2) -> column 2 and (2,1) -> column 1. So that's two different permutations. So coloring three cells here allows two perfect matchings. So k=3 is not good.What if we color two cells? Let's say color a and d. Then the only perfect matching is a and d. If we color a and c, then there's only one perfect matching: a and c. Wait, but in that case, if you color cells a and c, which are in the same column (column 1). Wait, no. In a 2x2 grid, a is (1,1), b is (1,2), c is (2,1), d is (2,2). If we color a and c, both in column 1. Then you can't place two tokens without overlapping in column 1. So that's impossible. So coloring two cells in the same column would not allow a perfect matching. So we need the colored cells to form a bipartite graph that has at least one perfect matching, and exactly one.So, for n=2, to have a unique perfect matching, the colored cells must form a graph where there's only one way to match rows to columns. For example, if we color a diagonal: a and d. Then there is exactly one perfect matching: a and d. If we color a, c, and d, then there are two perfect matchings: a and d, or c and d. Wait, no. If a, c, d are colored, then in the first row, you can pick a or d. If you pick a, then second row can pick d. If you pick d in the first row, then second row can pick c. So two perfect matchings. So that's three cells again. So if we color two cells: a and d. Then only one perfect matching. Similarly, if we color b and c, then only one perfect matching. So maximum k here is 2. So for n=2, maximum k is 2.But wait, what if we color a, b, d. Then, first row has a and b. Second row has d. So in first row, if you pick a, then second row picks d. If you pick b in first row, second row can't pick anything because only d is colored, but column 2 is already taken. Wait, no: second row has only d colored, which is column 2. So if first row picks b (column 2), then second row must pick d (column 2), which is conflicting. So actually, only one perfect matching: a and d. So k=3 here gives unique perfect matching. Wait, but this contradicts my previous thought. Let me check again.If colored cells are a(1,1), b(1,2), d(2,2). So first row: a and b. Second row: d. Now, in the first row, if you choose a (column 1), then second row must choose d (column 2). If you choose b (column 2) in the first row, then second row must choose d, but d is in column 2, which is already used. So only one perfect matching: a and d. Therefore, k=3 is possible for n=2. But earlier, when I colored a, b, c, there were two perfect matchings. So it depends on which three cells you color.Wait, so in the case where we color a, b, d, there's a unique perfect matching. So k=3 is possible. Then why did I think before that k=3 is not possible? Because it depends on the arrangement. So perhaps for n=2, the maximum k is 3?Wait, let's test that. Suppose we color three cells: a, b, d. Then, as above, only one perfect matching. But if we color a, c, d, then two perfect matchings. So it's possible to color three cells and still have a unique perfect matching. So maybe the answer for n=2 is 3? But that contradicts my previous thought.Wait, this is confusing. Let's check again. Let's take n=2:Case 1: color a, b, d. Then:- Row 1: a, b (columns 1, 2)- Row 2: d (column 2)Possible selections:- For row 1, choose a (column 1), then row 2 must choose d (column 2). That's one permutation.- For row 1, choose b (column 2), then row 2 can't choose anything, because the only colored cell is d (column 2), which is already taken. So only one perfect matching.So yes, with three cells colored, you can have a unique perfect matching.Similarly, if you color a, c, d:- Row 1: a (column 1)- Row 2: c (column 1), d (column 2)But row 1 can only pick a, then row 2 must pick d. If row 1 picks a, column 1 is taken, so row 2 can't pick c. So only one perfect matching. Wait, but c is in column 1, so if row 1 picks a (column 1), row 2 can't pick c. So in this case, coloring a, c, d would also result in only one perfect matching. Wait, but that's not right. Wait, row 2 has two colored cells: c (column 1) and d (column 2). If row 1 picks a (column 1), then row 2 can't pick c (column 1), so must pick d (column 2). If row 1 had picked... but row 1 can only pick a. So only one perfect matching. Hmm. Wait, so coloring three cells in different configurations can still result in unique perfect matching.Wait, but in this case, if we color a, c, d, then row 1 has only a colored. So row 1 must pick a. Then row 2 can pick d. So only one perfect matching. Similarly, if we color a, b, d, row 1 can pick a or b. If it picks b, then row 2 can't pick anything. Wait, no. If row 1 picks b (column 2), then row 2 must pick a cell in column not 2. But row 2 only has d (column 2) colored. So row 2 can't pick anything. Therefore, only a and d is possible. So in both cases, three colored cells can result in unique perfect matchings.But wait, this seems like in n=2, you can color three cells and have a unique perfect matching. But that contradicts my initial thought. So maybe for n=2, the maximum k is 3? But then, if we color all four cells (k=4), there are two perfect matchings. So maximum k is 3?But the question says "Aya wants to color k cells so that there is a unique way to place n tokens". So for n=2, the maximum k is 3? Hmm. But then, how does that generalize?Wait, let's test n=3. Maybe we can see a pattern.Suppose n=3. Let's try to color as many cells as possible such that there's a unique perfect matching.One way to get a unique perfect matching is to have a structure that forces the permutation. For example, if in each row, there's only one cell that can be chosen without conflicting. But that would be a diagonal, which gives k=n. But maybe we can have more cells.Wait, in the n=2 case, we colored three cells (which is n(n-1) +1 = 2*1 +1=3). Wait, n(n-1) +1. For n=2, that's 3. For n=3, that would be 7. Maybe the formula is n(n-1) +1. Let me check.Alternatively, the maximum k could be n(n-1) +1. Wait, for n=1, that would be 1(0)+1=1. For n=2, 2*1 +1=3. For n=3, 3*2 +1=7. So maybe the answer is n(n-1)+1. But let's verify.Wait, in n=3, if we color 7 cells, can we have a unique perfect matching? How?Wait, suppose we have a diagonal plus all cells except one in each row. Wait, maybe not. Let's think.Alternatively, think of a graph that is a "tree" plus some edges. Wait, no, in bipartite graphs, trees are acyclic, but perfect matchings in trees... Not sure.Alternatively, consider that for a unique perfect matching, the graph must be such that it contains a permutation matrix, and all other cells are such that they don't form an alternating cycle or something. Wait, I remember that a bipartite graph has a unique perfect matching if and only if it is a so-called "matching-covered" graph with no alternating cycles. But maybe there's a more straightforward characterization.Wait, here's a theorem: A bipartite graph has exactly one perfect matching if and only if it can be oriented such that it forms a directed cycle-free graph with all edges directed from one partition to the other, except for the perfect matching edges which are directed in the opposite direction. But I might be misremembering.Alternatively, there's a result that a bipartite graph with a unique perfect matching must have a certain structure, like being a "bi-arborescence" or something. Hmm, not sure.Alternatively, think recursively. For the unique perfect matching, suppose you have a cell that's the only one in its row or column. Then, you must choose that cell in the perfect matching, and then the problem reduces to the remaining (n-1)x(n-1) grid.If we can keep having cells that are forced to be in the perfect matching because they're the only ones in their row or column, then we can build up the graph in such a way. So, for example, if you have a cell that's the only one in its row, then you have to pick it, then in the remaining grid, same thing.But if we want to maximize the number of colored cells, then maybe in each step, we don't force a cell until the last possible moment. Wait, maybe.Alternatively, consider that in order to have a unique perfect matching, the graph must be such that it is a union of a permutation matrix and a lower triangular matrix, or something like that. For instance, in the case of n=2, coloring the diagonal and the lower triangle (three cells) allows only one perfect matching.Wait, let's visualize. For n=2, if we color (1,1), (2,1), and (2,2). So, first row has (1,1), second row has (2,1) and (2,2). Then, the perfect matching must choose (1,1) in the first row, then in the second row, you can choose either (2,1) or (2,2). Wait, that's two perfect matchings. So that's not unique.But if we color (1,1), (1,2), and (2,2). Then, first row has two options, but choosing (1,2) forces the second row to pick (2,2), but that's column 2, which is already used. Wait, no. If first row picks (1,2), then second row must pick (2,1), but (2,1) isn't colored. So only one perfect matching: (1,1) and (2,2). So that works. So three cells.Similarly, for n=3, if we have the main diagonal colored, plus all cells above the diagonal, or something. Wait, maybe not. Let's think.Suppose we have a diagonal matrix, so each row has exactly one cell on the diagonal. That gives a unique perfect matching, with k=n. But we can add more cells as long as they don't introduce another perfect matching.For example, in n=3, if we color the diagonal (3 cells) plus all cells in the upper triangle (3 more cells, total k=6). Wait, let's see. If we have the diagonal (1,1), (2,2), (3,3) and also (1,2), (1,3), (2,3). Then, can we have multiple perfect matchings?In the first row, we have (1,1), (1,2), (1,3). If we pick (1,1), then in the second row, we have (2,2), (2,3). If we pick (2,2), then third row must pick (3,3). If we pick (2,3) in the second row, then third row must pick (3,3), but column 3 is already taken. Wait, no, third row has only (3,3) colored. So, if second row picks (2,3), then third row can't pick anything. So actually, only one path: (1,1), (2,2), (3,3). Alternatively, if first row picks (1,2), then column 2 is taken. Second row has (2,2) (taken) and (2,3). So pick (2,3). Then third row must pick (3,3), but column 3 is taken. So invalid. Similarly, first row picks (1,3), then column 3 taken. Second row picks (2,2), then third row must pick (3,3), column 3 is taken. So again, only one perfect matching.Wait, so even with k=6 (for n=3), we can have a unique perfect matching. But wait, 6 is n(n-1). For n=3, n(n-1) +1 would be 7. But in this case, k=6 gives unique perfect matching. So maybe my initial thought was wrong.Wait, but let's check. If we have the diagonal plus all cells above the diagonal, that's n(n+1)/2. For n=3, that's 6 cells. But in this case, as above, the only perfect matching is the diagonal. So k=6 allows unique perfect matching. But n(n-1) +1 would be 7. So perhaps this is a different structure.Wait, maybe there's a different way to color the cells. For example, if we color the diagonal and all cells below the diagonal. Then, in each row, you have the diagonal cell and all cells to the left. Then, when trying to form a perfect matching, you have to choose the diagonal cell in each row, because if you choose a cell to the left, then in the subsequent rows, you can't complete the matching. Wait, let's check for n=3.Suppose colored cells are:Row 1: (1,1)Row 2: (2,1), (2,2)Row 3: (3,1), (3,2), (3,3)So, k=1+2+3=6. Then, the perfect matching must be (1,1), (2,2), (3,3). Because if in row 1 we pick (1,1), then in row 2, we can pick (2,2), leaving row 3 to pick (3,3). If in row 2, we pick (2,1), then column 1 is taken, so row 3 must pick (3,2) or (3,3). But if row 3 picks (3,2), column 2 is still available. Wait, but then we would have (1,1), (2,1), which is same column. Wait, no. Wait, if row 1 picks (1,1), column 1 is taken. Then row 2 can't pick (2,1), so must pick (2,2). Then row 3 picks (3,3). Alternatively, if row 1 picks something else, but row 1 only has (1,1). So only one perfect matching. Wait, no. Wait, in this structure, each row i has cells from column 1 to column i. So row 1: column 1; row 2: columns 1,2; row 3: columns 1,2,3. Then, the only perfect matching is the diagonal. Because in row 1, you have to pick (1,1). Then in row 2, you have to pick (2,2), since column 1 is already taken. Then in row 3, pick (3,3). So this gives a unique perfect matching with k=6 for n=3. So that works.Similarly, if you have a lower triangular matrix (including the diagonal), then you have k=n(n+1)/2, which for n=3 is 6. But this is more than n(n-1) +1=7- wait, for n=3, n(n-1)+1 is 3*2+1=7, which is more than 6. So which is correct?Wait, in the lower triangular matrix, you have k= n(n+1)/2. For n=3, that's 6. But n(n-1)+1 for n=3 is 7. So which one is possible? Can we color 7 cells in a 3x3 grid and still have a unique perfect matching?Let me try. Let's take the lower triangular matrix (6 cells) and add one more cell. Let's say we add cell (1,2). So now, row 1 has (1,1) and (1,2). Rows 2 and 3 remain as before. Now, is the perfect matching still unique?In row 1, we can choose (1,1) or (1,2). If we choose (1,1), then proceed as before: (2,2) and (3,3). If we choose (1,2), then column 2 is taken. Then in row 2, can't choose (2,2), so must choose (2,1). Then in row 3, columns 1 and 2 are taken, so must choose (3,3). So now, we have two perfect matchings: (1,1), (2,2), (3,3) and (1,2), (2,1), (3,3). So uniqueness is lost. Therefore, adding that cell breaks the uniqueness.Alternatively, if we add a different cell. Let's say we add cell (3,2) which is already in the lower triangular. Wait, it's already there. So maybe add cell (2,3). Let's see. Original lower triangular matrix has (2,1), (2,2). Adding (2,3). So row 2 now has (2,1), (2,2), (2,3). Now, in row 1, pick (1,1). Then in row 2, pick (2,2) or (2,3). If we pick (2,3), then in row 3, we have to pick (3,1) or (3,2). But column 3 is taken. Wait, row 3 has (3,1), (3,2), (3,3). If column 3 is taken by (2,3), then row 3 can pick (3,1) or (3,2). Suppose row 3 picks (3,1). Then the perfect matching is (1,1), (2,3), (3,1). But that's a different permutation. So uniqueness is lost. Hence, adding that cell also creates another perfect matching.Alternatively, add cell (3,1). But it's already there. Hmm. So perhaps adding any cell to the lower triangular matrix introduces another perfect matching. Therefore, the maximum k for n=3 is 6, which is n(n+1)/2. But wait, this contradicts the previous thought where for n=2, the lower triangular matrix gives k=3, which is 2(2+1)/2=3. For n=2, that works. For n=3, 3(4)/2=6. So perhaps the formula is k= n(n+1)/2. But wait, when n=1, that gives 1(2)/2=1, which is correct. For n=2, 3, which we saw is possible. For n=3, 6. But wait, the problem is asking for the maximum k. So is the maximum k= n(n+1)/2? But let's check n=3 with k=7.Wait, n=3, k=7. If we color all cells except one. Suppose we remove one cell from the full grid. Then, does that leave a unique perfect matching? If we remove a cell that is part of every perfect matching, then the remaining grid has none. But if we remove a cell that is not in any perfect matching, then the number of perfect matchings remains the same. If we remove a cell that is in some but not all perfect matchings, then the number decreases. But will it become unique?For example, in a 3x3 grid, if we remove cell (1,1). Then, how many perfect matchings are left? Originally, there are 6 perfect matchings (3!). After removing (1,1), how many? The first row now has two cells: (1,2) and (1,3). For each of these choices:If we pick (1,2), then column 2 is taken. Then in the remaining 2x2 grid (rows 2-3, columns 1,3), there are two perfect matchings: (2,1) and (3,3); or (2,3) and (3,1). Similarly, if we pick (1,3) in the first row, then column 3 is taken. The remaining 2x2 grid (rows 2-3, columns 1,2) also has two perfect matchings. So total of 2 + 2 = 4 perfect matchings. So removing one cell can reduce the number, but not to one.Alternatively, remove a different cell. Suppose we remove (2,2). Then, how many perfect matchings? Let's see. In a 3x3 grid without (2,2). The possible perfect matchings:Row 1 can be (1,1), (1,2), (1,3).Similarly for other rows. But since (2,2) is removed, row 2 has cells (2,1), (2,3).Let me count the number of perfect matchings. This might be complicated. Alternatively, use inclusion-exclusion. But maybe it's easier to realize that removing a single cell from a complete bipartite graph K_{3,3} leaves a graph that still has multiple perfect matchings.Therefore, simply removing one cell from the complete grid won't give uniqueness. Hence, the maximum k is not n^2 -1.So perhaps the maximum k is indeed n(n+1)/2, achieved by the lower triangular matrix. But wait, for n=3, that's 6. Let me verify again if that works.In the lower triangular matrix:Row 1: (1,1)Row 2: (2,1), (2,2)Row 3: (3,1), (3,2), (3,3)So the colored cells are all on or below the diagonal. Then, the only perfect matching is the diagonal. Because:- In row 1, only (1,1) is available.- In row 2, after choosing (1,1), column 1 is taken, so must choose (2,2).- In row 3, after choosing (1,1) and (2,2), columns 1 and 2 are taken, so must choose (3,3).Thus, unique perfect matching. Similarly, for n=3, k=6 works.But what if we add another cell? Say, in row 1, column 2. Then, row 1 has (1,1) and (1,2). Now, if we choose (1,2) in row 1, column 2 is taken. Then in row 2, we can't choose (2,2), so we have to choose (2,1). Then in row 3, columns 1 and 2 are taken, so must choose (3,3). So another perfect matching: (1,2), (2,1), (3,3). Thus, uniqueness is lost. Therefore, adding any cell above the diagonal in the lower triangular matrix allows another perfect matching.Therefore, the maximum k for n=3 is indeed 6, which is n(n+1)/2. Similarly, for n=2, it's 3=2*3/2. For n=1, 1=1*2/2. So perhaps the general formula is k= n(n+1)/2. But wait, the problem states "n >=1". For n=1, maximum k=1, which matches 1(1+1)/2=1. For n=2, 3, which matches. For n=3, 6. So this seems to hold.But wait, in the n=2 case, if we use the lower triangular matrix, we have k=3, which allows a unique perfect matching. If we use a different structure, like coloring three cells but not the lower triangle, can we have more cells? Wait, no. For n=2, the lower triangular matrix is the maximum possible, which is 3. Similarly, for n=3, it's 6. So the pattern is the triangular number.But the question is asking for the maximum k where it's achievable. So the answer would be k= n(n+1)/2.But wait, let me verify with another example. Take n=4. The lower triangular matrix would have 4(4+1)/2=10 cells. Then, the perfect matching would have to be the diagonal: (1,1), (2,2), (3,3), (4,4). Because in each row, you have to pick the diagonal since the others are blocked by previous choices. Let's see:Row 1: only (1,1) if we follow strict lower triangular. Wait, no. Wait, lower triangular matrix for n=4 would have:Row 1: (1,1)Row 2: (2,1), (2,2)Row 3: (3,1), (3,2), (3,3)Row 4: (4,1), (4,2), (4,3), (4,4)But if you structure it as lower triangular including the diagonal, then each row i has columns 1 through i. So in this case, the only perfect matching is the diagonal because:- Row 1 must pick (1,1)- Row 2, column 1 is taken, so must pick (2,2)- Row 3, columns 1 and 2 are taken, so must pick (3,3)- Row 4, columns 1,2,3 are taken, so must pick (4,4)Thus, unique perfect matching. And k=10 for n=4.If we add any extra cell above the diagonal in some row, say in row 1, column 2, then:- Row 1 can pick (1,1) or (1,2)If it picks (1,2), then:- Row 2 must pick (2,1) (since column 2 is taken)- Row 3 must pick (3,3)- Row 4 must pick (4,4)So another perfect matching: (1,2), (2,1), (3,3), (4,4). Hence, uniqueness lost.Therefore, the maximum k is indeed the triangular number n(n+1)/2.But wait, the problem says "the maximum value of k for which Aya's wish is achievable". So if for each n, the maximum k is n(n+1)/2, then that's the answer. But let me check literature or standard results.Wait, I recall that in bipartite graphs, the maximum number of edges with a unique perfect matching is the triangular number. This is sometimes known as the "maximal uniquely matchable" graph. Yes, in bipartite graphs, the maximal such graph is indeed the lower triangular matrix with k = n(n+1)/2. Therefore, the answer should be boxed{frac{n(n+1)}{2}}.But wait, wait. Let me confirm once more with n=3. If we have the lower triangular matrix (6 cells), and we add another cell, say (1,2), making k=7. Then, as we saw earlier, there are two perfect matchings. So k=7 is too much. Similarly, in n=2, k=3 is the maximum. Therefore, the pattern holds.Therefore, the maximum k is n(n+1)/2.Final AnswerThe maximum value of ( k ) is (boxed{frac{n(n+1)}{2}}).</think>To determine the maximum value of ( k ) for which Aya can color ( k ) cells in an ( n times n ) table such that there is a unique way to place ( n ) tokens on the colored cells without any two tokens being in the same row or column, we need to consider the structure of the colored cells. The key insight is that the colored cells must form a bipartite graph with a unique perfect matching. A known result in graph theory indicates that the maximal bipartite graph with a unique perfect matching is structured as a lower triangular matrix (including the diagonal). This structure ensures that each row ( i ) contains cells from column 1 to column ( i ), forcing a unique perfect matching by sequentially eliminating column choices for subsequent rows.For an ( n times n ) table, the number of cells in such a lower triangular matrix (including the diagonal) is given by the triangular number:[frac{n(n+1)}{2}]This structure ensures that there is only one way to place ( n ) tokens such that each token is in a unique row and column. Adding any cell above the diagonal would introduce multiple perfect matchings, thus breaking the uniqueness.Thus, the maximum value of ( k ) for which Aya's wish is achievable is:[boxed{frac{n(n+1)}{2}}],<think>Okay, so I need to figure out the maximum number of cells, k, that Aya can color in an n x n table such that there's a unique way to place n tokens on the colored cells without any two tokens being in the same row or column. Hmm, this seems related to permutations and maybe something like Latin squares? Let me think.First, let me make sure I understand the problem correctly. We have an n x n grid. Aya colors k cells. Then, we need to place n tokens such that each token is in a different row and column (like a permutation matrix), and all tokens are on colored cells. The key is that there must be only one such arrangement possible. And we want the maximum k where this uniqueness is achievable.So, if k is too large, there might be multiple ways to place the tokens. If k is as large as possible while still having only one unique placement, that's the number we need. Let me start with small n to see if I can spot a pattern.Case n=1: The table is 1x1. To have a unique placement, the only cell must be colored. So k=1, which is n. But maybe maximum k here is 1, which is n. That works.Case n=2: A 2x2 table. If we color all 4 cells, then there are 2 possible permutations (the identity and the swap), so that's bad. If we color 3 cells, let's see. Suppose we color the diagonal cells (1,1), (2,2) and another one, say (1,2). Then possible placements: (1,1) and (2,2), or (1,2) and (2,1). But (2,1) isn't colored, so only the diagonal is possible. Wait, if we color (1,1), (1,2), (2,2), then the possible placements are (1,1) and (2,2), or (1,2) and (2,1). But (2,1) is not colored, so only the first option is possible. So in this case, k=3 works. Is there a way to color 4 cells? No, because then both permutations would be possible. So maximum k for n=2 is 3.Wait, but 3 is n(n-1) + 1? For n=2, 2*1 +1=3. Hmm. Maybe the formula is n(n-1) +1. Wait, let's check for n=3.n=3: If the formula holds, maximum k would be 3*2 +1=7. Let's see. Suppose we color all cells except one. Let's say we remove one cell not on the main diagonal. Then, is the permutation unique? Maybe not. Wait, perhaps it's more structured.Wait, in n=2 case, coloring three cells where only one permutation is possible. For n=3, maybe color all cells except those in a single row or column. Wait, but if we remove a whole row or column, that might interfere with having a permutation. Wait, no, because you need exactly one per row and column. So if a row has only one cell colored, then that's forced. Hmm, maybe building up the colored cells such that each row and column has exactly one cell that is uniquely determined.Alternatively, think of a permutation matrix, which has n cells (one per row and column). If we color those n cells, then there's exactly one way to place the tokens. But k=n here, which is minimal. But we want the maximum k where uniqueness still holds. So the question is, how many cells can we add to a permutation matrix such that no other permutation is possible.Wait, that might be the key. Start with a permutation matrix (n cells), then add cells in such a way that no other permutation can be formed. Each added cell should not create an alternate permutation. So, the problem reduces to adding as many cells as possible to a permutation matrix without creating another permutation.So, what's the maximum number of cells you can have in a bipartite graph (rows and columns as two partitions, cells as edges) such that there's exactly one perfect matching. We need the maximum number of edges with exactly one perfect matching.This is a known problem in combinatorics. The maximum number of edges in a bipartite graph with a unique perfect matching. For an n x n bipartite graph, the maximum number of edges is n(n-1)/2 + n. Wait, no. Wait, in the case of n=2, the maximum is 3, which is 2*1 +1=3, as before. For n=3, if the formula is n(n-1)/2 +n, that would be 3*2/2 +3=3+3=6, but we saw that maybe 7 is possible? Wait, maybe not. Let me check.Wait, actually, in bipartite graphs, the maximum number of edges with a unique perfect matching is n(n-1)/2 +1. Wait, I need to recall some theorems. There's a theorem by Hetyei (1964) for bipartite graphs with a unique perfect matching. The maximum number of edges is n(n-1)/2 +1. Wait, let's check for n=2. Then 2(1)/2 +1=1+1=2, but we have n=2 case where 3 edges work. Hmm, that contradicts. So maybe I got the theorem wrong.Alternatively, perhaps in bipartite graphs, the maximum number of edges with exactly one perfect matching is n^2 - (n-1)^2 +1? Not sure. Let me think differently.Suppose we have a permutation matrix (n cells). To prevent any other permutation, we need to block all possible alternate permutations. Each alternate permutation is a cycle. For example, in a permutation matrix, if you have a 2x2 cycle, you can swap two elements. To prevent that, we need to ensure that in every possible cycle, at least one cell is missing. So, if we add cells to the permutation matrix, but in such a way that every possible cycle has at least one cell missing. But how to maximize the number of added cells. For example, in the case of n=2, the permutation matrix is the diagonal. If we add the cell (1,2), then the other permutation (2,1) is blocked because (2,1) is not colored. Wait, but if we add both (1,2) and (2,1), then we can have both permutations. So to prevent that, we can only add one of them. Wait, but if we add (1,2) and (2,2), as in the n=2 case, then it's still unique. Wait, maybe the key is to have a structure where in each row after the first, the number of choices is constrained by previous choices.Alternatively, think of the permutation as being forced to follow a specific path. If you have a triangular shape, maybe. For example, in the case of n=3, color all cells on or above the main diagonal. Then, the only permutation is the identity permutation. Because in the first row, you can only choose the first column. Then in the second row, you can only choose the second column, since the first is already taken, and the third row can only choose the third. But if you color all cells on or above the diagonal, that's n(n+1)/2 cells. For n=3, that's 6 cells. But is the only permutation the identity? Let's see. For n=3, if we have cells (1,1), (1,2), (1,3), (2,2), (2,3), (3,3). Then, possible permutations: (1,1), (2,2), (3,3) is one. But can we have another permutation? For example, (1,2), (2,3), (3,1). But (3,1) is not colored. So that's invalid. Or (1,3), (2,1), (3,2). But (2,1) and (3,2) aren't colored. So the only permutation is the identity. So in this case, k=6 for n=3, which is n(n+1)/2=6. But wait, n(n+1)/2 is 6 for n=3. However, if this is the case, then the maximum k would be n(n+1)/2. But wait, for n=2, that formula gives 3, which matches. For n=1, it gives 1, which is correct. So maybe the maximum k is n(n+1)/2.But wait, let's test for n=3. If we color all cells on or above the diagonal, then the number of colored cells is 6. As we saw, only the identity permutation is possible. So k=6 works. But can we color more cells? Suppose we add one more cell below the diagonal. Let's say (2,1). Now, can we have another permutation? Let's see. If (2,1) is colored, then possible permutation could be (1,1), (2,1), (3,3). But (2,1) is in column 1, which conflicts with (1,1). So no. Wait, permutations need one per row and column. So to have another permutation, you need to have a different set of cells. For example, (1,2), (2,1), (3,3). But (1,2) and (2,1) are colored (assuming we added (2,1)), but (3,3) is colored. So that would be a valid permutation. But (1,2) is colored, (2,1) is colored, (3,3) is colored. So that's a valid permutation. Therefore, adding (2,1) allows another permutation. Therefore, the maximum k is indeed 6 for n=3. If we add any cell below the diagonal, we create the possibility of another permutation. Therefore, the upper triangular matrix with all cells on or above the diagonal gives a unique permutation and has k= n(n+1)/2. But wait, let me check again for n=3. If we color the upper triangular including diagonal, then can we have another permutation? For example, (1,2), (2,3), (3,1). But (3,1) is not colored. Similarly, (1,3), (2,1), (3,2). Again, (2,1) and (3,2) are not colored. The only cells colored are on or above the diagonal. So any permutation other than the identity would require selecting a cell below the diagonal, which is not colored. Therefore, the only permutation is the identity. Therefore, this works. Thus, for n=3, k=6 is possible, which is n(n+1)/2.Similarly, for n=1, k=1, which is 1(2)/2=1. For n=2, 2(3)/2=3, which works. So this seems to hold. So perhaps the maximum k is n(n+1)/2. However, is this the maximum possible?Wait, but let me think if there's a way to color more cells. Suppose for n=3, instead of coloring the upper triangular, we color all cells except one in each row and column in a way that still forces the identity permutation. For example, maybe a more efficient structure. But if you color more than n(n+1)/2, which is 6 for n=3, you would have to color at least 7 cells. Let's try. Suppose we color all cells except (3,1). Then, the colored cells are all except (3,1). Let's see if there's a unique permutation. The identity permutation is possible. Is there another one? For example, permuting rows 1 and 2: (1,2), (2,1), (3,3). But (1,2) and (2,1) are colored, (3,3) is colored. So that's a valid permutation. Therefore, uniqueness is lost. So coloring 8 cells (all except one) allows multiple permutations. Therefore, 8 is too many. Similarly, if you color 7 cells, say remove two cells. Let's remove (3,1) and (3,2). Then, in row 3, only (3,3) is colored. So in that case, column 3 is forced. Then in row 1 and 2, we have all cells except (3,1) and (3,2). Wait, no. If we remove (3,1) and (3,2), then in row 3, only (3,3) is colored. So column 3 is taken by row 3. Then in rows 1 and 2, columns 1 and 2 are available. But if in rows 1 and 2, all cells are colored except maybe some. Wait, if we remove (3,1) and (3,2), then in rows 1 and 2, columns 1 and 2 are fully colored? If we colored all except (3,1) and (3,2), then rows 1 and 2 have all their cells colored. So in rows 1 and 2, columns 1 and 2 are available. So the submatrix for rows 1,2 and columns 1,2 is fully colored. Therefore, there are two possible permutations in that submatrix, combined with row 3. Therefore, total permutations would be 2. So uniqueness is lost. Therefore, even removing two cells might not be sufficient. Therefore, perhaps the upper triangular matrix is indeed the maximum. Because in that structure, each row i has exactly i cells colored (from column 1 to column i). Wait, no. Wait, in the upper triangular matrix, row 1 has n cells, row 2 has n-1, ..., row n has 1 cell. Wait, no, actually, in an upper triangular matrix (including diagonal), row 1 has n cells, row 2 has n-1 cells (columns 2 to n), but no. Wait, upper triangular matrix would have row 1: columns 1 to n, row 2: columns 2 to n, etc. But no, that's not right. Actually, an upper triangular matrix has entries where column >= row. So row 1 has columns 1 to n, row 2 has columns 2 to n, ..., row n has column n. Therefore, the number of cells is n + (n-1) + ... + 1 = n(n+1)/2. So that's correct.In such a matrix, the only permutation possible is the identity permutation. Because in row 1, you can choose any column, but if you choose a column j >1, then in row j, you must choose a column >= j, but column j is already taken by row 1, so you can't. Wait, actually, let's see. Suppose we try to make a permutation different from the identity. Let's say in row 1, we choose column 2. Then, in row 2, we can't choose column 2, so we have to choose column 3 or higher. Suppose we choose column 3. Then in row 3, we have to choose column 4 or higher, but in an upper triangular matrix, row 3 only has columns 3 to n. If n=3, row 3 only has column 3. So if we chose column 3 in row 2, then row 3 has nowhere to go. Therefore, such a permutation is impossible. Therefore, the only possible permutation is the identity. Thus, the upper triangular matrix with diagonal works. Therefore, the maximum k is n(n+1)/2. But let me check for n=3 again. If we have 6 cells colored as upper triangular, can we have another permutation? Let's try. Suppose we try (1,2), (2,3), (3,1). But (3,1) is not colored (since in upper triangular, column must be >= row, so row 3 only has column 3). So invalid. Similarly, (1,3), (2,1), (3,2). (2,1) and (3,2) are not colored. So no. Any permutation other than the identity requires selecting a cell below the diagonal, which isn't colored. Therefore, only the identity permutation is possible. Therefore, this construction works. And the number of cells is n(n+1)/2, which for n=1,2,3 gives 1,3,6, which are the correct maximums. So perhaps the answer is n(n+1)/2. But wait, the question is from Aya's perspective. She wants to color k cells such that there's a unique way to place n tokens on colored cells with no two in the same row or column. So the maximum k is n(n+1)/2. But I need to verify whether this is indeed the maximum. Is there a way to color more cells?Suppose for n=3, can I color 7 cells and still have a unique permutation? Let's see. Suppose I color all cells except (3,1) and (2,1). Then, in row 3, column 3 is the only colored cell. In row 2, columns 2 and 3 are colored. In row 1, all columns are colored. So the permutation must have token in (3,3). Then in row 2, columns 2 or 3. But column 3 is taken by row 3, so row 2 must choose column 2. Then row 1 can choose column 1. So the only permutation is (1,1), (2,2), (3,3). However, the number of colored cells here is 3 (row1:3 cells) + 2 (row2:2 cells) +1 (row3:1 cell) =6. Wait, but if we remove only two cells, (3,1) and (2,1), then total colored cells are 9 - 2 =7. Wait, rows:Row1: 3 cells.Row2: columns 2,3 (since (2,1) is removed). So 2 cells.Row3: column 3 (since (3,1) is removed). So 1 cell.Total: 3 +2 +1=6. Wait, but 9 -2=7. There's a discrepancy here. Wait, no, actually, if in a 3x3 grid, you remove two cells, you have 7 cells left. But according to the row counts above, it's 3 +2 +1=6. That doesn't add up. Wait, maybe I made a mistake.Wait, if we remove (2,1) and (3,1), then:Row1: all 3 cells.Row2: columns 2 and 3 (two cells).Row3: columns 2 and 3 (since we only removed column 1). Wait, no. If we remove (3,1), then row3 has columns 2 and 3. Wait, original grid has each row with columns 1,2,3. So removing (2,1) from row2 leaves columns 2,3. Removing (3,1) from row3 leaves columns 2,3. Therefore, row1: 3 cells, row2:2 cells, row3:2 cells. Total colored cells: 3+2+2=7.But in this case, the permutation must be:Row3 must choose column 3, because if row3 chooses column 2, then row2 must choose column3, but then row1 can choose column1. Wait:Option 1: row3 chooses column3. Then row2 must choose column2. Then row1 chooses column1. That's the identity permutation.Option 2: row3 chooses column2. Then row2 must choose column3. Then row1 chooses column1. So permutation (1,1), (2,3), (3,2). But (3,2) is colored (since we only removed column1 from row3). So this is another permutation. Therefore, uniqueness is lost. So even though we removed two cells, we still have multiple permutations. Therefore, this doesn't work.Therefore, coloring 7 cells allows multiple permutations. Therefore, the upper triangular matrix with 6 cells is indeed the maximum for n=3.Therefore, it seems that the maximum k is n(n+1)/2. This is achieved by coloring all cells on or above the main diagonal. This construction ensures that the only permutation possible is the identity permutation. Any attempt to color more cells would allow for another permutation by creating cycles that can be swapped, hence violating uniqueness.But wait, let's consider another structure. Suppose we have a permutation matrix where each row after the first has only one additional cell colored. Wait, maybe like a lower triangular matrix. But similar reasoning applies. The key is that the structure must force the permutation to follow a specific path, leaving no alternatives.Alternatively, if we color cells such that there's only one permutation, similar to a directed acyclic graph with a unique topological order. The upper triangular matrix ensures that each step is forced: in the first row, you can choose any column, but if you don't choose column1, then subsequent rows can't compensate. But actually, in the upper triangular matrix, the first row has all columns colored. If you choose column2 in row1, then row2 can't choose column2, so must choose column3, but then row3 can't choose column3, and so on. However, in the upper triangular case, row i can only choose columns >=i. So if you choose column j >i in row i, then row j must choose a column >=j, but column j is already taken, leading to a conflict. Hence, the only safe choice is to pick column i in row i, leading to the identity permutation.Therefore, this structure enforces the identity permutation as the only possibility. Hence, the maximum k is n(n+1)/2.Another way to see it: To have a unique permutation, the graph must be such that it's a DAG with a unique perfect matching, which in the case of the upper triangular matrix, it's a DAG with edges only from lower indices to higher, forcing the permutation to be the identity.Therefore, after considering examples and structures, I conclude that the maximum k is n(n+1)/2. Final AnswerThe maximum value of ( k ) is (boxed{frac{n(n+1)}{2}}).</think>To determine the maximum value of ( k ) for which Aya can color ( k ) cells in an ( n times n ) table such that there is a unique way to place ( n ) tokens on the colored cells without any two tokens being in the same row or column, we need to ensure that the configuration of colored cells allows only one perfect matching (permutation).Key steps and reasoning:1. Understanding the Problem: We need to color cells such that there is a unique permutation matrix (one token per row and column) possible. This requires that no alternate permutations can be formed using the colored cells.2. Small Cases Analysis: - For ( n = 1 ), the only cell must be colored, so ( k = 1 ). - For ( n = 2 ), coloring three cells (e.g., upper triangular matrix) results in a unique permutation, so ( k = 3 ). - For ( n = 3 ), coloring six cells (upper triangular matrix) results in a unique permutation, so ( k = 6 ).3. General Case: - The structure that ensures a unique permutation is an upper triangular matrix (including the main diagonal). This structure forces the identity permutation as the only possible solution because any deviation would require selecting a cell below the diagonal, which is not colored. - The number of cells in an upper triangular matrix (including the diagonal) for an ( n times n ) table is given by the sum of the first ( n ) natural numbers: ( frac{n(n+1)}{2} ).4. Verification: - Coloring cells in an upper triangular matrix ensures that each row ( i ) can only choose columns ( geq i ). Any attempt to choose a column ( j > i ) leads to conflicts in subsequent rows, thus enforcing the identity permutation. - Adding any cells below the diagonal would allow alternate permutations, hence violating uniqueness.Thus, the maximum value of ( k ) for which Aya's wish is achievable is (boxed{frac{n(n+1)}{2}}).]
🔑:Solution 1 First, Aya can choose the diagonal, which we will call D, consisting of the cells with coordinates (x, x). Aya then proceeds to color all the cells located on D or above D. Step by step, we observe that she has no other choice but to place her n tokens on the diagonal D itself. Furthermore, by doing so, she colors 1+2+cdots+n=n(n+1) / 2 cells. Thus, k geqslant n(n+1) / 2.Conversely, consider a coloring of k cells that satisfies the conditions of the problem, and the only possible arrangement of the tokens on these k cells. Even if we exchange any two rows of the table, our coloring will still satisfy the conditions of the problem. Therefore, and without loss of generality, we assume that the tokens are on the diagonal D by successively exchanging rows of the table.We then group the cells outside of D into pairs: each cell with coordinates (x, y) is paired with the cell that is symmetric to it with respect to D, i.e., the cell with coordinates (y, x). If our coloring contained two cells of the same pair, say the cells (x, y) and (y, x), Aya could have removed the tokens from the cells (x, x) and (y, y) and placed them in the cells (x, y) and (y, x). Therefore, our coloring contains at most one cell from each pair. Since the diagonal has n cells and the other cells are distributed into n(n-1) / 2 pairs, we have thus colored at most n+n(n-1) / 2=n(n+1) / 2 cells, so that k leqslant n(n+1) / 2.We conclude that k=n(n+1) / 2.
❓:Calculate the result of left(-7right)+3.A: -4B: -10C: -21D: 4
🧠:[<think>Okay, so I have this math problem here: (-7) + 3. And the options are A: -4, B: -10, C: -21, D: 4. Hmm, let me think. Alright, adding a positive number to a negative number. I remember that when you add a positive number to a negative number, you can think of it as moving to the right on the number line if the positive number is smaller than the absolute value of the negative one. Wait, maybe I should break it down more carefully.First, the problem is (-7) plus 3. So starting at -7 on the number line, adding 3 would mean moving 3 units to the right. Let's visualize that. If I'm at -7, moving 3 steps towards zero. So -7 + 1 is -6, then -6 +1 is -5, and -5 +1 is -4. So after adding 3, I end up at -4. That would be answer A.Alternatively, maybe I can think of it as subtracting their absolute values. Since one is negative and the other is positive. The larger absolute value here is 7, and the smaller is 3. So subtract 3 from 7, which gives 4, and since the larger number was negative, the result is negative. So -7 +3 = -(7-3) = -4. Yeah, that also gives A.Wait, another way. Let me recall the rules for adding integers with different signs. The rule says to subtract the smaller absolute value from the larger one and take the sign of the number with the larger absolute value. So here, |-7| = 7 and |3| = 3. Subtract 3 from 7, get 4. Since 7 is larger and it's negative, the result is negative 4. So A is correct.But let me check again. Maybe I made a mistake. If I have a debt of 7 dollars and then I earn 3 dollars, how much do I have? So I still owe 4 dollars. That's -4. Makes sense. So A is the right answer.Looking at the options: A is -4. The other options are B: -10, which would be if you added the absolute values and kept the negative, but that's for when both numbers are negative. C: -21, which is multiplying them maybe? But that's not the case here. D: 4, which is if you subtract 7 - 3 and take positive, but no, since the signs are different. So yeah, definitely A.</think>To calculate ((-7) + 3):1. Number Line Approach: Start at (-7) and move (3) units to the right: [ -7 rightarrow -6 rightarrow -5 rightarrow -4 ] Result: (-4).2. Absolute Value Subtraction: Subtract the smaller absolute value from the larger one and keep the sign of the larger: [ |!-7!| - |3| = 7 - 3 = 4 quad text{(sign of the larger absolute value: negative)} ] Result: (-4).3. Real-world Context: A debt of (7) reduced by (3) leaves a debt of (4), which is (-4).All methods confirm the answer is (boxed{A}).,<think>Okay, let's see. I need to calculate (-7) + 3. Hmm, adding a positive number to a negative number. Alright, so starting at -7 on the number line, if I add 3, I should move three units to the right. Let me visualize that. Starting at -7, moving one step right is -6, another to -5, and a third to -4. So that should be -4. Let me check another way. The absolute values are 7 and 3. The difference between them is 4, and since 7 is larger than 3, and the original number was negative, the result should be -4. Yeah, that matches. So the answer should be A: -4. Wait, but let me make sure I didn't mix up the signs. If I have a negative and a positive, the result takes the sign of the number with the larger absolute value. Right, because 7 is bigger than 3, so the negative sign wins, and 7-3 is 4, so -4. Yep, that's correct. I think A is the right choice here.</think>To calculate ((-7) + 3):1. Start at (-7) on the number line.2. Move (3) units to the right: (-7 rightarrow -6 rightarrow -5 rightarrow -4).3. The result is (-4), as the larger absolute value ((7)) determines the sign (negative).Answer: boxed{A}]
🔑:To calculate the result of left(-7right)+3, we follow these steps:1. Start with the given expression: left(-7right)+3.2. Recognize that adding 3 to -7 is the same as subtracting 3 from 7, but keeping the negative sign: =-left(7-3right).3. Perform the subtraction inside the parentheses: 7-3 = 4.4. Apply the negative sign: =-4.Therefore, the final answer, following the rules and encapsulating it, is boxed{-4}. This corresponds to choice boxed{A}.
❓:## Problem StatementCalculate the definite integral:int_{0}^{2 pi} sin ^{8}left(frac{x}{4}right) d x
🧠:[<think>Okay, so I need to calculate the definite integral of sin^8(x/4) from 0 to 2π. Hmm, integrating powers of sine can be tricky. Let me think about how to approach this. I remember that for even powers of sine or cosine, we can use power-reduction formulas or maybe expand using multiple angle identities. Let me recall the steps.First, since the exponent is 8, which is a large even number, using the power-reduction formula multiple times might be the way to go. Alternatively, I could use the binomial theorem with the complex exponential form of sine, but that might get complicated. Let me see.The integrand is sin^8(x/4). Let me make a substitution first to simplify the integral. Let’s set t = x/4. Then, dt = dx/4, so dx = 4 dt. When x goes from 0 to 2π, t goes from 0 to π/2. Wait, hold on: x is from 0 to 2π, so t = x/4 would be from 0 to (2π)/4 = π/2. So the integral becomes 4 ∫₀^{π/2} sin^8(t) dt. Hmm, okay, that might make things a bit simpler, but not by much. The main challenge is still integrating sin^8(t) over 0 to π/2, then multiplied by 4.Alternatively, maybe integrating over 0 to 2π directly. Wait, but the substitution to t = x/4 changes the interval. Let me check that again. If x is from 0 to 2π, then t = x/4 is from 0 to 2π/4 = π/2. So yes, the limits are from 0 to π/2. Therefore, the integral becomes 4 times the integral from 0 to π/2 of sin^8(t) dt.But maybe there's a better substitution or identity? Let me recall that the integral of sin^{2n}(t) over 0 to π/2 is given by the Beta function or the Gamma function? There is a formula for ∫₀^{π/2} sin^{2n}t dt = (π/2) * ( (2n)! ) / (4^n (n!)^2 ) ). Is that right? Let me verify.Yes, the formula for the integral of sin^{2n}x dx from 0 to π/2 is (π/2) * ( (2n)! ) / (4^n (n!)^2 ). So if n = 4 here, since we have sin^8(t), which is sin^{2*4}t. So plugging n=4 into that formula gives (π/2) * (8!)/(4^4 (4!)^2 ). Then, the integral we have is 4 times that. So the entire integral would be 4 * (π/2) * (40320)/(256 * (24)^2 ). Let me compute that step by step.First, compute 8! which is 40320. Then 4^4 is 256. Then 4! is 24, so (4!)^2 is 24*24=576. Therefore, the expression becomes (π/2) * 40320 / (256 * 576). Let me compute the denominator: 256*576. 256*500=128000, 256*76=19,456. So total is 128,000 + 19,456 = 147,456. Wait, but 256*576: let me compute 256*576.Alternatively, 256*576 = 256*(500 + 76) = 256*500 + 256*76. 256*500 = 128,000. 256*70 = 17,920. 256*6=1,536. So 17,920 + 1,536 = 19,456. Then total is 128,000 + 19,456 = 147,456. So denominator is 147,456.Numerator is 40,320. So 40,320 / 147,456. Let me simplify this fraction. Let's divide numerator and denominator by 48. 40,320 ÷48= 840. 147,456 ÷48=3,072. So 840/3,072. Divide numerator and denominator by 24: 840 ÷24=35, 3,072 ÷24=128. So 35/128. Therefore, the fraction simplifies to 35/128.So the integral from 0 to π/2 is (π/2)*(35/128) = (35π)/256. Then multiply by 4 from earlier substitution: 4*(35π)/256 = (140π)/256. Simplify this fraction. 140 and 256 are both divisible by 4: 140 ÷4=35, 256 ÷4=64. So it's 35π/64. Therefore, the definite integral ∫₀^{2π} sin^8(x/4) dx is 35π/64.Wait, let me check if I did the substitution correctly. The original substitution was t = x/4, so dx = 4 dt. When x goes from 0 to 2π, t goes from 0 to π/2. Then the integral becomes 4 ∫₀^{π/2} sin^8(t) dt. Then using the formula for ∫₀^{π/2} sin^{2n}t dt = (π/2)*( (2n)! ) / (4^n (n!)^2 ). So n=4 here. Then (2n)! = 8! = 40320. 4^n = 256. (n!)^2 = (24)^2=576. So (40320)/(256*576) = 40320/147456 = 35/128 as before. Then (π/2)*(35/128) = 35π/256. Multiply by 4 gives 35π/64. That seems right.Alternatively, another way to compute ∫ sin^8(t) dt is using power-reduction formulas multiple times. Let me try that method to verify.We know that sin^2θ = (1 - cos2θ)/2. So sin^8θ = (sin²θ)^4 = [(1 - cos2θ)/2]^4. Let me expand this:[(1 - cos2θ)/2]^4 = (1/16)(1 - cos2θ)^4. Then expand (1 - cos2θ)^4 using binomial theorem:1 - 4cos2θ + 6cos²2θ - 4cos³2θ + cos^42θ.Therefore, sin^8θ = (1/16)[1 - 4cos2θ + 6cos²2θ - 4cos³2θ + cos^42θ].Now, integrating term by term from 0 to π/2. But integrating cos terms over integer multiples over 0 to π/2 might result in zeros for odd powers. Wait, but we have cos2θ, cos²2θ, cos^32θ, and cos^42θ. Let's check each term.First term: ∫₀^{π/2} 1 dθ = π/2.Second term: -4 ∫₀^{π/2} cos2θ dθ. The integral of cos2θ from 0 to π/2 is [sin2θ/2] from 0 to π/2. At π/2: sin(π) = 0. At 0: sin0=0. So this integral is 0. Therefore, second term is 0.Third term: 6 ∫₀^{π/2} cos²2θ dθ. Use power-reduction again: cos²2θ = (1 + cos4θ)/2. So integral becomes 6*(1/2) ∫₀^{π/2} (1 + cos4θ) dθ = 3 [ ∫₀^{π/2}1 dθ + ∫₀^{π/2}cos4θ dθ ].First integral: π/2. Second integral: ∫cos4θ dθ from 0 to π/2 is [sin4θ/4] from 0 to π/2. At π/2: sin(2π) = 0. At 0: sin0=0. So integral is 0. Thus, third term contributes 3*(π/2) = 3π/2.Fourth term: -4 ∫₀^{π/2} cos^32θ dθ. The integral of cos^3 is going to involve some reduction. But since the interval is over 0 to π/2, and we have an odd power of cosine. Wait, cos^32θ. Let’s note that cos^3 is an odd function if we shift appropriately, but over 0 to π/2, let's compute it.Alternatively, use substitution. Let u = 2θ, then du = 2 dθ, so dθ = du/2. The limits become 0 to π. So integral becomes -4 * ∫₀^{π} cos^3u * (du/2) = -2 ∫₀^{π} cos^3u du. Now, integral of cos^3u over 0 to π. Since cos^3u is an even function around π/2? Wait, over 0 to π, cos^3u is symmetric. Wait, but cos^3u is actually negative from π/2 to π. Let me compute it directly.∫cos^3u du = ∫cosu*(1 - sin^2u) du. Let’s set v = sinu, dv = cosu du. So integral becomes ∫(1 - v^2) dv = v - v^3/3 + C. Evaluated from 0 to π: [sinπ - (sinπ)^3/3] - [sin0 - (sin0)^3/3] = (0 - 0) - (0 - 0) = 0. So the integral of cos^3u from 0 to π is 0. Therefore, fourth term is -2*0 = 0.Fifth term: ∫₀^{π/2} cos^42θ dθ. Again, use power-reduction. Let’s set φ = 2θ, so dφ = 2dθ, dθ = dφ/2. Limits become 0 to π. Then the integral becomes (1/2) ∫₀^{π} cos^4φ dφ. Now, integrate cos^4φ over 0 to π.Using power-reduction formula: cos^4φ = (cos²φ)^2 = [(1 + cos2φ)/2]^2 = (1 + 2cos2φ + cos²2φ)/4. Then integrate term by term:∫cos^4φ dφ = ∫(1/4 + (1/2)cos2φ + (1/4)cos²2φ) dφ.Integrate from 0 to π:First term: (1/4)π.Second term: (1/2) ∫₀^{π} cos2φ dφ = (1/2)*(sin2φ/2)|₀^{π} = (1/4)(sin2π - sin0) = 0.Third term: (1/4) ∫₀^{π} cos²2φ dφ. Use power-reduction again: cos²2φ = (1 + cos4φ)/2. So integral becomes (1/4)*(1/2) ∫₀^{π} (1 + cos4φ) dφ = (1/8)[π + (sin4φ)/4 |₀^{π}] = (1/8)π + 0 = π/8.So total integral: (1/4)π + 0 + π/8 = (2π/8 + π/8) = 3π/8. Therefore, ∫₀^{π} cos^4φ dφ = 3π/8. Then back to original substitution: (1/2)*(3π/8) = 3π/16.So the fifth term is 3π/16.Putting all together, the original integral sin^8θ from 0 to π/2 is (1/16)[ first term + third term + fifth term ].Wait, no. Wait: sin^8θ was expanded as (1/16)[1 - 4cos2θ + 6cos²2θ -4cos³2θ + cos^42θ]. Then when we integrate term by term from 0 to π/2:Integral = (1/16)[ ∫1 dθ -4∫cos2θ dθ +6∫cos²2θ dθ -4∫cos³2θ dθ + ∫cos^42θ dθ ]From previous calculations:∫1 dθ = π/2-4∫cos2θ dθ = 06∫cos²2θ dθ = 6*(π/2)*(1/2) + 6*(0) = 3π/2? Wait, no. Wait earlier in the third term, we found that ∫cos²2θ dθ from 0 to π/2 is (1/2) ∫cos²φ dφ from 0 to π, where φ = 2θ. Wait, maybe I need to re-examine that.Wait, let's go back. When we computed the third term, which was 6 ∫₀^{π/2} cos²2θ dθ, we used substitution u = 2θ, so integral becomes 6*(1/2) ∫₀^{π} cos²u du. Then ∫cos²u du from 0 to π is (π/2). So 6*(1/2)*(π/2) = 6*(π/4) = 3π/2. Wait, but that contradicts the previous step. Wait, no, maybe in that step, when we used the substitution φ=2θ, then the integral over 0 to π/2 becomes 1/2 the integral over 0 to π. Then ∫cos²u du from 0 to π is (π/2). So yes, ∫cos²2θ dθ from 0 to π/2 is (1/2)(π/2) = π/4. Then 6 times that is 6*(π/4) = 3π/2. Wait, but in the earlier approach, when expanding cos²2θ as (1 + cos4θ)/2, we had 3*(π/2). Wait, perhaps I made a miscalculation there.Wait, in the third term:We had 6 ∫₀^{π/2} cos²2θ dθ. Let's use substitution u = 2θ, du = 2dθ, so dθ = du/2. Limits from 0 to π. Then integral becomes 6*(1/2) ∫₀^{π} cos²u du. The integral of cos²u over 0 to π is (π/2). Therefore, the integral becomes 6*(1/2)*(π/2) = 6*(π/4) = 3π/2. Therefore, third term is 3π/2.Fourth term was -4 ∫cos³2θ dθ, which we found to be 0.Fifth term: ∫cos^42θ dθ. Let's use substitution u=2θ, du=2dθ, dθ=du/2. Limits 0 to π. Integral becomes (1/2) ∫₀^{π} cos^4u du. From previous calculation, ∫cos^4u du from 0 to π is 3π/8. Therefore, (1/2)*(3π/8) = 3π/16. So fifth term is 3π/16.Therefore, putting all together:Integral = (1/16)[ π/2 + 0 + 3π/2 + 0 + 3π/16 ]Compute each term:First term: π/2Third term: 3π/2Fifth term: 3π/16Total inside the brackets: π/2 + 3π/2 = 2π; 2π + 3π/16 = (32π/16 + 3π/16) = 35π/16.Therefore, integral = (1/16)*(35π/16) = 35π/256. Which matches the previous result.Then, multiply by 4 (from substitution t = x/4, dx = 4dt) gives 4*(35π/256) = 140π/256 = 35π/64. So both methods lead to the same result. That’s reassuring.Alternatively, another approach could be to use the beta function. The integral ∫₀^{π/2} sin^{2n}t dt = (sqrt(π)/2) * Gamma(n + 1/2)/Gamma(n + 1). For n=4, this becomes (sqrt(π)/2) * Gamma(9/2)/Gamma(5). Gamma(9/2) is (7/2)(5/2)(3/2)(1/2)sqrt(π) = (105/16)sqrt(π). Gamma(5) is 4! =24. Therefore, the integral becomes (sqrt(π)/2) * (105 sqrt(π)/16)/24 = (sqrt(π)/2)*(105 sqrt(π))/(16*24). Multiply sqrt(π) * sqrt(π) = π. So this is (π/2)*(105)/(16*24) = (π/2)*(105)/(384) = (105π)/(768). Simplify 105/768: divide numerator and denominator by 3: 35/256. Thus, (35π)/256. Which again matches. Then multiplied by 4 gives 35π/64. So all methods confirm that the integral is 35π/64.Therefore, I can be confident that the answer is 35π/64. Let me just check once more for calculation errors. For instance, when expanding (1 - cos2θ)^4, the coefficients are 1, -4, 6, -4, 1. Then integrating each term, as we did. The calculation of the integrals for each term seems correct. The only possible error could be in the expansion or in the substitution steps. But all methods lead to the same result, so it's likely correct.Another sanity check: for the integral of sin^8(x/4) over 0 to 2π. The function sin^8(x/4) has a period of 8π, since sin(x/4) has period 8π, so raising it to the 8th power doesn't change the period. But we are integrating over 0 to 2π, which is less than the period. However, when we did the substitution t = x/4, we ended up integrating over 0 to π/2, which is a quarter of the period of sin(t). However, since we are using power-reduction formulas and the integral over the full period would be different, but here we’re integrating over a part of the period. But through substitution, we adjusted the limits correctly, and the formula for the integral from 0 to π/2 is still applicable. So I think it's okay.Alternatively, if we had integrated over the full period of sin^8(t), which is π (since sin^8(t) has period π, because sin(t) has period 2π, and when you square it, it becomes π, so sin^8(t) would have period π/2? Wait, no. Wait, sin(t) has period 2π. sin^2(t) has period π, since sin(t + π) = -sin t, so sin²(t + π) = sin²t. Similarly, sin^4(t) also has period π, and sin^8(t) also has period π. Wait, actually, even exponents don't change the period. Wait, sin(t) has period 2π. sin(kt) has period 2π/k. But when you square sin(kt), you get sin²(kt) which has period π/k. Similarly, sin^8(kt) would have period π/k. In our case, the original integrand is sin^8(x/4). So sin(x/4) has period 8π. Then sin^8(x/4) has period 4π. So integrating over 0 to 2π is half the period. But even so, the substitution to t = x/4 changes variables to integrate over 0 to π/2, which is a quarter of the original period (8π). Wait, perhaps the period of sin^8(x/4) is actually 4π, as mentioned. But integrating over 0 to 2π is half of that. However, when we perform the substitution, we get the integral over 0 to π/2 of sin^8(t) dt, multiplied by 4. So the key is that we are integrating over a part of the period. But using the power-reduction formula, which works regardless of the interval, as long as the integrals of cosine terms over that interval are computed correctly. Since we did term-by-term integration and found that several terms vanish, it's still valid.Therefore, the final answer is 35π/64.Final AnswerThe definite integral is boxed{dfrac{35pi}{64}}.</think>To calculate the definite integral (int_{0}^{2 pi} sin ^{8}left(frac{x}{4}right) d x), we start by making a substitution (t = frac{x}{4}), which transforms the integral. This substitution gives (dt = frac{dx}{4}) or (dx = 4 dt). The limits of integration change from (x = 0) to (x = 2pi) into (t = 0) to (t = frac{pi}{2}). The integral then becomes:[4 int_{0}^{pi/2} sin^8(t) , dt]We use the formula for the integral of (sin^{2n}(t)) over ([0, pi/2]):[int_{0}^{pi/2} sin^{2n}(t) , dt = frac{pi}{2} cdot frac{(2n)!}{4^n (n!)^2}]For (n = 4), we calculate the necessary factorials and simplify:[int_{0}^{pi/2} sin^8(t) , dt = frac{pi}{2} cdot frac{8!}{4^4 (4!)^2}]Calculating the factorials and simplifying:[8! = 40320, quad 4! = 24, quad 4^4 = 256, quad (4!)^2 = 576][frac{8!}{4^4 (4!)^2} = frac{40320}{256 cdot 576} = frac{40320}{147456} = frac{35}{128}]Thus, the integral becomes:[frac{pi}{2} cdot frac{35}{128} = frac{35pi}{256}]Multiplying by 4 (from the substitution):[4 cdot frac{35pi}{256} = frac{140pi}{256} = frac{35pi}{64}]Therefore, the definite integral is:[boxed{dfrac{35pi}{64}}],<think>Okay, so I need to calculate the definite integral of sin^8(x/4) from 0 to 2π. Hmm, integrating powers of sine can be tricky. Let me think about how to approach this. I remember that for even powers of sine or cosine, we can use the power-reduction formulas from trigonometry. Maybe I can expand sin^8(theta) into a series of cosines with multiple angles. Let me recall the formula for sin^{2n}(theta). First, let me make a substitution to simplify the integral. The argument inside the sine is x/4, so maybe let’s set t = x/4. Then, when x = 0, t = 0, and when x = 2π, t = 2π/4 = π/2. So, the integral becomes:∫₀^{π/2} sin^8(t) * 4 dtBecause dx = 4 dt. So, the integral is 4 times the integral of sin^8(t) from 0 to π/2. That might be a simpler interval to work with.Now, I need to compute ∫ sin^8(t) dt. As I thought earlier, power-reduction formulas. Let me remember the formula for sin^{2n}(t). The general approach is to use the identity sin^2(theta) = (1 - cos(2theta))/2, and then apply it recursively for higher powers.So, sin^8(t) can be written as [sin^2(t)]^4 = [(1 - cos(2t))/2]^4. Let me expand that.First, expand (1 - cos(2t))^4. Let's compute that:(1 - cos(2t))^4 = 1 - 4cos(2t) + 6cos^2(2t) - 4cos^3(2t) + cos^4(2t)Then divide by 2^4 = 16. So, sin^8(t) = [1 - 4cos(2t) + 6cos^2(2t) - 4cos^3(2t) + cos^4(2t)] / 16But integrating term by term from 0 to π/2. Let's split the integral into five separate integrals:I = (1/16) [ ∫1 dt - 4∫cos(2t) dt + 6∫cos²(2t) dt - 4∫cos³(2t) dt + ∫cos⁴(2t) dt ] evaluated from 0 to π/2Hmm, this seems manageable. Let's compute each integral separately.First term: ∫1 dt from 0 to π/2 is just π/2.Second term: ∫cos(2t) dt from 0 to π/2. The integral of cos(2t) is (1/2)sin(2t). Evaluated from 0 to π/2: (1/2)[sin(π) - sin(0)] = (1/2)(0 - 0) = 0. So the second term is zero.Third term: ∫cos²(2t) dt. Use the power-reduction formula again: cos²(x) = (1 + cos(2x))/2. So, cos²(2t) = (1 + cos(4t))/2. Therefore, the integral becomes:∫(1 + cos(4t))/2 dt = (1/2)∫1 dt + (1/2)∫cos(4t) dtCompute from 0 to π/2:First part: (1/2)(π/2) = π/4Second part: (1/2)*(1/4)sin(4t) evaluated from 0 to π/2. At π/2: sin(4*(π/2)) = sin(2π) = 0; at 0: sin(0) = 0. So, the second part is zero. Thus, the third term integral is π/4.Fourth term: ∫cos³(2t) dt. Hmm, integrating odd powers of cosine. For odd powers, we can use substitution. Let’s set u = sin(2t), then du = 2cos(2t) dt. But here we have cos³(2t) = cos²(2t)*cos(2t) = (1 - sin²(2t))cos(2t). So:∫cos³(2t) dt = ∫(1 - sin²(2t))cos(2t) dt. Let u = sin(2t), du = 2cos(2t) dt, so (1/2)du = cos(2t) dt. Then the integral becomes:(1/2) ∫(1 - u²) du = (1/2)(u - u³/3) + C = (1/2)(sin(2t) - sin³(2t)/3) + CNow evaluate from 0 to π/2:At π/2: sin(2*(π/2)) = sin(π) = 0; sin³(π)/3 = 0At 0: sin(0) - sin³(0)/3 = 0. So, the entire integral is (1/2)(0 - 0) - (1/2)(0 - 0) = 0. Therefore, the fourth term is zero.Fifth term: ∫cos⁴(2t) dt. Again, power-reduction formula. cos⁴(2t) = [cos²(2t)]² = [(1 + cos(4t))/2]^2 = (1 + 2cos(4t) + cos²(4t))/4Then integrate:∫cos⁴(2t) dt = ∫(1 + 2cos(4t) + cos²(4t))/4 dt = (1/4)∫1 dt + (1/4)*2∫cos(4t) dt + (1/4)∫cos²(4t) dtSimplify:(1/4)∫1 dt = (1/4)t(1/2)∫cos(4t) dt = (1/2)*(1/4)sin(4t) = (1/8)sin(4t)(1/4)∫cos²(4t) dt. Again, use power-reduction: cos²(4t) = (1 + cos(8t))/2. Therefore:(1/4)*(1/2)∫(1 + cos(8t)) dt = (1/8)∫1 dt + (1/8)∫cos(8t) dtSo overall:∫cos⁴(2t) dt = (1/4)t + (1/8)sin(4t) + (1/8)t + (1/64)sin(8t) + CCombine terms:(1/4 + 1/8)t = (3/8)tThe sine terms: (1/8)sin(4t) + (1/64)sin(8t)So, evaluated from 0 to π/2:First, the t term: (3/8)*(π/2) = 3π/16The sine terms: At π/2: sin(4*(π/2)) = sin(2π) = 0; sin(8*(π/2)) = sin(4π) = 0. At 0: sin(0) and sin(0) = 0. So, the sine terms contribute nothing. Therefore, the integral of cos⁴(2t) from 0 to π/2 is 3π/16.Putting all the terms back together:Original integral I = (1/16)[π/2 - 4*0 + 6*(π/4) - 4*0 + 3π/16] Let me compute each coefficient:First term: (1/16)*(π/2) = π/32Third term: 6*(π/4) = (3π)/2, multiplied by (1/16) is 3π/32Fifth term: (3π/16)*(1/16) = 3π/256Wait, hold on. Wait, the fifth term is ∫cos⁴(2t) dt which is 3π/16, so multiplied by (1/16) gives (3π/16)*(1) = 3π/16*1? Wait, let me check again.Wait, the integral I is:I = (1/16)[ ∫1 dt -4∫cos(2t) dt +6∫cos²(2t) dt -4∫cos³(2t) dt + ∫cos⁴(2t) dt ]So, the coefficients are:First term: 1/16 * π/2Second term: -4/16 * 0 = 0Third term: 6/16 * π/4Wait, hold on, no. Wait, the entire integral is broken into:[Integral1 - 4*Integral2 + 6*Integral3 - 4*Integral4 + Integral5], each integral computed from 0 to π/2, then multiplied by 1/16.So, Integral1 is π/2, Integral2 is 0, Integral3 is π/4, Integral4 is 0, Integral5 is 3π/16.Therefore, plugging in:I = (1/16)[π/2 - 4*0 + 6*(π/4) - 4*0 + 3π/16]Compute each term:π/2 = 8π/166*(π/4) = 6π/4 = 3π/2 = 24π/163π/16 remains as is.So, adding them up:8π/16 + 24π/16 + 3π/16 = (8 + 24 + 3)π/16 = 35π/16Multiply by 1/16:35π/(16*16) = 35π/256Therefore, the integral of sin^8(t) from 0 to π/2 is 35π/256.But remember, the original substitution gave us:Original integral = 4 * ∫₀^{π/2} sin^8(t) dt = 4*(35π/256) = 35π/64Wait, let me check that arithmetic:4*(35π/256) = (4/1)*(35π/256) = (140π)/256 = Simplify numerator and denominator by dividing numerator and denominator by 4: 35π/64. Yes.So, the integral from 0 to 2π of sin^8(x/4) dx is 35π/64.Wait, but let me verify if all the steps are correct. Let me double-check the coefficients when expanding (1 - cos2t)^4.Expanding (1 - cos2t)^4:Using binomial expansion:1^4 = 1-4*1^3*cos2t = -4cos2t+6*1^2*(cos2t)^2 = +6cos²2t-4*1*(cos2t)^3 = -4cos³2t+ (cos2t)^4 = +cos⁴2tSo that part is correct. Divided by 16, so sin^8(t) = [1 -4cos2t +6cos²2t -4cos³2t +cos⁴2t]/16. Correct.Then integrating term by term. The integral over 0 to π/2 of cos2t dt. The antiderivative is (1/2)sin2t. Evaluated from 0 to π/2: sin(π) - sin(0) = 0. So, integral of cos2t is zero. Same for integral of cos³2t. The integral of cos²2t we converted to π/4. Integral of cos⁴2t was 3π/16. Let me verify that again.For cos⁴2t, we used power-reduction twice. First, cos²(2t) = (1 + cos4t)/2, then squared that. Then expanded and integrated. The integral became 3π/16. Let me recompute that.Wait, ∫cos⁴2t dt from 0 to π/2:First, we had:cos⁴2t = [ (1 + cos4t)/2 ]² = (1 + 2cos4t + cos²4t)/4Integrate term by term:(1/4)∫1 dt + (1/2)∫cos4t dt + (1/4)∫cos²4t dtFirst integral: (1/4)(π/2) = π/8Second integral: (1/2)*(1/4)sin4t evaluated from 0 to π/2. At π/2: sin4*(π/2) = sin2π = 0. At 0: sin0 = 0. So, zero.Third integral: (1/4)∫cos²4t dt. Then, cos²4t = (1 + cos8t)/2. So:(1/4)*(1/2)∫1 dt + (1/4)*(1/2)∫cos8t dt = (1/8)(π/2) + (1/8)*(1/8)sin8t evaluated from 0 to π/2.First part: (1/8)(π/2) = π/16Second part: (1/64)(sin4π - sin0) = 0.So total third integral: π/16Therefore, total ∫cos⁴2t dt = π/8 + 0 + π/16 = 3π/16. Correct.So that integral is 3π/16. Then, in the original expression:I = (1/16)[π/2 + 6*(π/4) + 3π/16]Wait, but the coefficients: [Integral1 -4*Integral2 +6*Integral3 -4*Integral4 +Integral5] is:Integral1: π/2-4*0: 0+6*(π/4): 6*(π/4) = 3π/2-4*0: 0+Integral5: 3π/16So total inside the brackets: π/2 + 3π/2 + 3π/16Convert to sixteenths:π/2 = 8π/163π/2 = 24π/163π/16 remains.Adding: 8π +24π +3π =35π over 16. So 35π/16. Then multiply by 1/16: 35π/256. Correct.Then multiply by 4 (from substitution) gives 35π/64. So the final answer is 35π/64.Hmm, let me check with another method. Maybe using the beta function or gamma function? For integrals of sin^n x over 0 to π/2, there is a formula:∫₀^{π/2} sin^n t dt = (sqrt(π)/2) * Γ((n+1)/2) / Γ((n+2)/2)But maybe I can use that to check.Given that n=8 here. So:∫₀^{π/2} sin^8 t dt = (sqrt(π)/2) * Γ(9/2)/Γ(5)But Γ(9/2) = (7/2)(5/2)(3/2)(1/2)sqrt(π) = (105/16)sqrt(π)Γ(5) = 4! = 24Thus,Integral = (sqrt(π)/2) * (105/16 sqrt(π)) /24 = (sqrt(π)/2) * (105 sqrt(π))/(16*24)Multiply terms:sqrt(π)*sqrt(π) = πSo,Integral = (π/2) * 105/(16*24) = (π/2) * 105/(384) = 105π/(768) = 35π/256Which matches our previous result. So this confirms that ∫₀^{π/2} sin^8 t dt = 35π/256, so the original integral is 4*(35π/256) = 35π/64.Therefore, the answer should be 35π/64.Final Answerboxed{dfrac{35pi}{64}}</think>To calculate the definite integral (int_{0}^{2 pi} sin ^{8}left(frac{x}{4}right) d x), we start by making a substitution (t = frac{x}{4}), which transforms the integral into (4 int_{0}^{pi/2} sin^8(t) , dt).Next, we use the power-reduction formula to expand (sin^8(t)):[sin^8(t) = left( frac{1 - cos(2t)}{2} right)^4 = frac{1 - 4cos(2t) + 6cos^2(2t) - 4cos^3(2t) + cos^4(2t)}{16}]We then integrate each term separately over the interval ([0, pi/2]):1. (int_{0}^{pi/2} 1 , dt = frac{pi}{2})2. (int_{0}^{pi/2} cos(2t) , dt = 0)3. (int_{0}^{pi/2} cos^2(2t) , dt = frac{pi}{4})4. (int_{0}^{pi/2} cos^3(2t) , dt = 0)5. (int_{0}^{pi/2} cos^4(2t) , dt = frac{3pi}{16})Combining these results, we have:[int_{0}^{pi/2} sin^8(t) , dt = frac{1}{16} left( frac{pi}{2} + 6 cdot frac{pi}{4} + frac{3pi}{16} right) = frac{1}{16} left( frac{8pi}{16} + frac{24pi}{16} + frac{3pi}{16} right) = frac{35pi}{256}]Multiplying by 4 (from the substitution (dx = 4 dt)), we get:[4 cdot frac{35pi}{256} = frac{35pi}{64}]Thus, the definite integral is:[boxed{dfrac{35pi}{64}}]]
🔑:## Solutionbegin{aligned}& int_{0}^{2 pi} sin ^{8}left(frac{x}{4}right) d x=left|sin ^{2}left(frac{x}{4}right)=frac{1}{2}left(1-cos left(frac{x}{2}right)right)right|=int_{0}^{2 pi} frac{1}{2^{4}}left(1-cos left(frac{x}{2}right)right)^{4} d x= & =frac{1}{2^{4}} cdot int_{0}^{2 pi}left(1-4 cos left(frac{x}{2}right)+6 cos ^{2}left(frac{x}{2}right)-4 cos ^{3}left(frac{x}{2}right)+cos ^{4}left(frac{x}{2}right)right) d x= & =frac{1}{2^{4}} cdot int_{0}^{2 pi}left(1-4 cos left(frac{x}{2}right)right) d x+frac{1}{2^{4}} cdot int_{0}^{2 pi} 6 cos ^{2}left(frac{x}{2}right) d x-frac{1}{2^{4}} cdot int_{0}^{2 pi} 4 cos ^{3}left(frac{x}{2}right) d x+frac{1}{2^{4}} cdot int_{0}^{2 pi} cos & =leftlvert, cos ^{2}left(frac{x}{2}right)=frac{1}{2}(1+cos x)right., cos ^{2}left(frac{x}{2}right)=1-sin ^{2}left(frac{x}{2}right), 2 dleft(sin left(frac{x}{2}right)right)=cos left(frac{x}{2}right) & =left.frac{1}{2^{4}} cdotleft(x-8 sin left(frac{x}{2}right)right)right|_{0} ^{2 pi}+frac{1}{2^{4}} cdot int_{0}^{2 pi} 6 cdot frac{1}{2}(1+cos x) d x-frac{1}{2^{4}} cdot int_{0}^{2 pi} 4left(1-sin ^{2}left(frac{x}{2}right)right) cdot 2 d(sin & +frac{1}{2^{4}} cdot int_{0}^{2 pi} frac{1}{2^{2}}(1+cos x)^{2} d x= & =frac{1}{2^{4}} cdot(2 pi-8 sin pi)-frac{1}{2^{4}} cdot(0-8 sin 0)+left.frac{1}{2^{4}} cdot 3(x+sin x)right|_{0} ^{2 pi}-frac{3}{2^{4}} cdot(0+0)-frac{1}{2} cdotleft(0-frac{1}{3} cdot 0^{3}right)+frac{1}{2} cdotleft(0-frac{1}{3} cdot 0^{3}right)+ & +frac{1}{2^{6}} cdot int_{0}^{2 pi}(1+2 cos x) d x+frac{1}{2^{6}} cdot int_{0}^{2 pi} cos ^{2} x d x= & =frac{pi}{8}+frac{3 pi}{8}-0+0+left.frac{1}{2^{6}} cdot(x+2 sin x)right|_{0} ^{2 pi}+frac{1}{2^{6}} cdot int_{0}^{2 pi} cos ^{2} x d x=left|cos ^{2} x=frac{1}{2}(1+cos 2 x)right|= & =frac{pi}{2}+frac{pi}{2^{5}}-0+frac{1}{2^{6}} cdot(2 pi+0)-frac{1}{2^{6}} cdot(0+0)+frac{1}{2^{6}} cdot int_{0}^{2 pi} frac{1}{2}left(1+cos ^{2 pi}right) d x=end{aligned}begin{aligned}& =frac{pi}{2}+frac{pi}{2^{5}}+frac{pi}{2^{6}}-0+left.frac{1}{2^{7}}left(x+frac{1}{2} sin 2 xright)right|_{0} ^{2 pi}=frac{pi}{2}+frac{pi}{2^{5}}+frac{1}{2^{7}}left(2 pi+frac{1}{2} cdot 0right)-frac{1}{2^{7}}left(0+frac{1}{2} cdot 0right)= & =frac{pi}{2}+frac{pi}{2^{5}}+frac{pi}{2^{6}}-0=frac{32 pi+2 pi+pi}{2^{6}}=frac{35 pi}{64}end{aligned}Source — «http://pluspi.org/wiki/index.php/������������_ %D0%9A%D1%83%D0%B7%D0%BD%D0%B5%D1%86%D0%BE%D0%B2_%D0%98%D0%BD%D1%82 % mathrm{D} 0 % mathrm{~B} 5 % mathrm{D} 0 % mathrm{~B} 3 % mathrm{D} 1 % 80 % mathrm{D} 0 % mathrm{~B} 0 % mathrm{D} 0 % mathrm{BB} % mathrm{D} 1 % 8 mathrm{~B}+10-16 % Categories: Kuznetsov's Problem Book Integrals Problem 10 mid IntegralsUkrainian Banner Network- Last edited: 16:01, June 1, 2009.- Content is available under CC-BY-SA 3.0.Created by GeeTeatoo## Problem Kuznetsov Integrals 10-17## Material from PlusPi